You are on page 1of 415
GATE SOLUTION IN MINING ENGINEERING (Containing Solved Questions 2015 - 2007) By RUPESH KUMAR SAHU LOVELY PRAKASHAN, DHANBAD idstone In immediate roof delays the local fallin goaf ofa coat mine, Under crushing of the pillars at outbye side is called (6) overriding of pitas. (0) spaling of pillars A back sight on a bench mark RL 100.00 m on the floor ofa tunnel is 3.25 m. The inverse staff Feading on a floor station of the tunnel is 1.25 m. The RL of the roof station in m is Answer: 104 to 105. & RLof the roof station [s = 100 + 3.25 41.25, 21085m The angle in degrees at which 2 ridge line intersects contours is (a) 0 (30 (45 (090 answer (D) Is prevented by 8 drum hoisting system through a vertical shaft, overwindl A) lly controller (0) detaching hook Caliper brake [D) safery catch nswer (8) he temperature ofa parcel of air decreases from 30.2°C to 28.9°C ast rises from an altitude of dm to 120%. The lpse rate forthe the atmosphere is \)subadiabatic (8) adiabatic (C)superadiabatic _—(0) transadiabatic newer (C) apse rate The rate at which atmospheric temperature decreases with increase n atitude uperadiabatic lapse rate ~This occurs when the atmosphere temperature drops more than ¥°C/100 m, he excess pore pressure in backfill material ina cut-and-filstope leads to a) reduction in the strength of the wall rock 3) enhancement of bearing strength of fil loss of shear resistance of fill D} prevention of progressive failure of erown pillar rnswer (C) ‘The primary purpose of eut holes for blasting an underground drivage's to {A) provide additional free face (6) have smooth surface after blasting (©) prevent over-breakage (0) reduce noise ‘Answer (A) 9 _Inatriangle Aac, the bearings ofthe sides AB, BC and CA are GO? 130°, and 270° ‘espectively. The interior angles A,B and Cin degrees respectively are (a) 110, 40,30 (8) 40, 110, 30 (€)30, 40, 110 (0) 30, 110, 40 Answer (0) Solution . 0° bearing of Al » bearing of BA= 60° + 180°= 240° and + bearing of BC= 130° + bearing of B= 130° + 180° =310° and * bearing of CA= 270° + bearing of AC = 270° ~ 180°= 90° Therefore , 2A bearing of AC~bearing of AB 24290" ~ 60°=30°, and 28 = bearing of 84 — bearing of BC 28 = 2408 ~ 1302 110°, and a 180° — 2B — 2A= 180° — 110° ~ 30° ee 40° 2.10 Ina binomial distribution, the probability of success p-> 0 and number of trials n> co such that 2 = np approaches toa finite value, The variance of the distribution is (A) npa (oyna (pa (a Answer {0} Solution . {mn binomial distribution : The varlance of random variable Xis given by var [x] = mp —p) Since, p-» 0,n> 00and.2-= np Therefore, Var [x] = a. —0) Var [x Q.11 Fora function f(x), itis given that f(0) 2 and f'(0 4. Ignoring al other higher order Aerivative terms, the value of (0.5) i= Answer 4, Solution Suppose that, f(x) = x4 + 4x42 When x =0, then f@)=2 Differentiating equation (1) wart, x, then F@)=4 44 When x =0, then f@=4 When 2 5, then from equation (1) f(OS) =05'+4x0542 fos) =4 Q.12 The two sides ofa parallelogram are glven by the vectors A = 26 3) and B area ofthe parallelogram is (a) 13 (0) 32 (10 Or Answer (A) Solution . bik axack 3 q| Is 2 ol AX B= 10 ~0) ~j(0-0) +K(4+9) axe 38 + Atea of parallelogram having two side vectors A and 8s given by = [A x B = [138] 3 2.13 Ina BOD test, ml of wastewaters diluted with pure water toll a 300 mil BOD bottle. The intial and final dissolved oxygen contents of the mix are 9.0 mg/l and 7.0 mg/l respectively. 5 ‘The BOD of wastewater, in me/! Is (ay (8)10 (120 (0) 600 Answer (C) Solution. Wehave, CxM =O: x Me (Or Sample BOD x Sample Volume = B00 Diluted x Volume Diluted ian State x Volume Diluted or 80 Sample 2.9. Depletion na/0, folume Diluted (ri “Sample Volume ty VO" hy © 800 Sample (met [+ D0. Depletion = 0.0. Initial 0.0. final or 0.0. 5 Day Sample BOD In mgft = O=20% 300 = 120me/t 14 A force of 501 is applied to.a wrench as shown inthe figure. The magnitude of the moment in N-mm of this force about the point Pis Answer : 7900 to 7920. Solution . | aus “Magnitude of moment about P= 50 x cas 30° x 200 ~ 50 x sin 30° x 30 = 7910 Nem Dilatancy of rock is associated with (A) increase in surface area after fragmentation (0) decrease in volume due to compression of rack (6) increase in shear strain due to cracking of rock (0) increase in volume due to cracking of rack Answer (0) 0.16 Abord and pilar panel having square pillars fs designed for 30% extraction during development. Ifthe gallery width cS m, the side ofthe pillar in m is. Answer: 25 to26, Solution . Extraction Ratio = or 7(a+5)? or 17 a? + 70+ ATS or 03a?-7a-175 2=25.68-2.28 Hence, pillar size 25.6 m. 2.17 Low shock and high gas pressure explosive is generally used forthe blasting of (a) hard and bitte rock mass (@)soft and joint rock mass (€) hacd and massive intact rock mass (0) soft and massive intact rock mass Answer (8) 18 The covariance of copper grade for a certain lag distance in an ore body is 6.0 ()* «the sis 10 (96), the semivariogram for the same lag distance in (96) is (yao (216.0 (o20 (064.0 Answer (A) Solution . We have, y(h) = 6 -0(h) Where, (hi) = semivariogram at lag distance h « 6(h) = covariance at lag distance h rn) 0 (06)? ~ 6 06)" 4/6 2/6 4/6 19 Thematina | ae 4s 2 26 476 4/6 (8) orthogonal (8) agonal (¢)shew-symmatyy (0) syminatic ‘Answer (A) Solution. 4/6 4/6 2/6| ze 416 ~a/6 . 10 9} aa'=lo 1 | oo 4 Aa =1 Q.20 gas mixture contains City, CoH, and. H, with respective concentrations of 75%, 15% ‘and 10% by volume. The lower explosibiity mit of CA, CaHgand Hy are5%, 3.3% and 4.2% respectively. The lower explosibility limit ofthe gas mixture, in percentage , is Answer # 1205. Solution . The lower explosbilty mit (1) for gas mixture i elven by: percent Where, a, Pa, Panos = contents in percent by volume, of each combustible component of the mie Na, Map Ma | «(pa lower explosion limit of each components X= 456% 21 Intake air containing 0.2% methane enters a section of an underground mine where ‘emission rate of methane 0.05 m?/s. Assuming thatthe threshold ini value of methane 's 1.25 %, the minimum quantity of fesh ar required in m/s is — Answer : 4610.49. Solution We have, Where, 4q= gas emission , m/sec Ca = gasin the intake air, percent Gp = permissible or safe maximum percent of gas in the return air 125-02 47 m/s 0.22 ina fully mechanized bord and pillar mining system, winning of coal and its transportation {rom the face is commonly earsied out withthe combination of 10 (A) Continuous miner shuttle car, feeder breaker and belt conveyor (6) Continuous miner, LHD, feeder breaker and chain conveyor (6) Continuous miner SDL feeder breaker and belt conveyor {0) Continuous miner, shuttle car, Feeder breaker and chain conveyor Answer (A) 0.23 Anunderground coal mine employing 1200 persons experiences 2 fatal injuri during the year 2013, The total injury rate per 1000 persons ‘employed for the year is, Answer: 13,010 13.6 Solution otal injury rate per 1000 persons employed = x 1000 = 134 24 Inself-contained chemical oxygen self rescuer, oxygen is produced by (A) Hopeatite (8) potassium peroxide {) sodium hydrox (0) Protosor Answer (8) 1.25 The fallure data ofan equipment follows an exponential distribution. Ifthe mean time between failures fs 3000 houts, the reliabilty ofthe equipment for 750 hours is Answer : 0.75 0! Solution . ‘he rlibity that the equipment run upto time t for exponential distribution is given by ~ where, u ean time between failure Sash per hous And t= 750 hours R(t) = e309 = 078 4.25 In.9.4.2 mwide and 3.0m high gallery in a coal seam, twolve shot holes are blaster per round, ‘The holes are charged with 2 explosive cartridge of 435 ¢ each. M the povider factor of the blasts 2.2 tonne/kg and specific gravity of coal is 1.4, the pull per round of blast in m is (145 (@)2.70 (3.30 (0)4.05, ‘Answer (C) Solution . we have, coal produced (conne) explosive required (kg) Powder factor = 428% pull 14 per round 122 per round 20m 27 The stadia reading with horizontal sight on a vertical staff held at 50 m froma tacheometer ‘are 1.285 and 1.780 m. The focal length of the objec glass is 25 em and the distance between ‘the object glass and the vertical axis ofthe tacheometer is 15 em. the stadia interval Answer: 2.48 102.52 Solution . 2 P sinsrumentstaton qa station Weknow, van conse =£ ‘Additive constant (C)= (fd) and ste Where, focaeath {stain nteral D~distance between instrument axs and objective s~staffintercept 8 Deksefed S0= kx0.49540.25+0.15 825 — 000249 m Smm 3 Ina shortwall panel, coal is extracted from the face by a continuous miner having rate of a2 production 30tonne/hour Coal having pec ray of. transported by stl cars of capacity 0.9 m? each to a feeder breaker located at 60m from the face. If the average speed of {he LHD 605 ms. and total aing an untadingte oF LAD i 405, he mumer of LAD reared to match the production of the continuous minis on 2 a ry ‘Answer (8) solution. Continuous miner production =304/n = 22 m/sec = 000595 m/sec speed = dotnet Time Traveltime =120 sec and X 120+ 40=280 sec Cyete time of Coal transported by LHD in one cycle time = 0,00595 280 1.67 m? No.of uo = 2 = 1.9 = 10.29, _Vertical photographs of an area lying 500m above the mean sea level are to be taken at a height scale of 1:20000 from an aircraft. I the camera has 2 focal length of 210 mm, the fyi ofthe aircraft above the mean sea evel inmis Answer £4700. “ Solution. ofeponse tc) hepa ir. | (A) P-2,0:3,R-1,5-4 (8) P-4,.Q-3R-4,5-2 15 (oP 2,0-4.R-453 (0)P2,03,n4,54 Answer (C) 31 Thevalueot i VI6—wFase (ayizs7 (0) soz4 (e252 (oats Answer (A) solution. Weave, 0.32 Arectangular field of area 20000 mt is to be divided in to 6 different plots by fencing as shown in the figure, The value of Lin m for which the total length of fencing becomes ‘minionurn is 16 Answer + 161 to 165. Sotation Let total feng length, Z = 7+ 3 Area of rectangular = W = 20000 2 za 4x 200434 a) piferemtng (Aw ttob for minimum value, Hence, 1633 0.33 Matenthe folowing fora ding system component Function oon 1. tation of ener a fragmenting rock @.bat rot 2. Reduction of energy le ut ernding revit bit 2. Conversion of rina for of energy in tomachanil nerey 5: Fusing mem 4 transmiston of energy from roe mover to applicator ()P3.01.025-4 (8) JOIR3S2; (C)PQ4RAS2; (0)P-2,04R3,5-4; Answer (€) ” 0.34 Forthe ventitation system shown, the combined resistance ofthe trunk airways and the shafts 2.2 Ns?m~®, The resistance ofthe spit A and 8 are 0.5 Nm" and 0.8 Ns*m ‘respectively, A regulator of size 2.0 misplaced in split A. Considering the fan generates a pressure of 1000 Pa, thea low in m/s in split B is Answer :10.2 t0 20.8, Dest Inaetrank sirway Solution Wehave, sit an sof eur ‘ivay 18 ANstm Total resistance of mine = 2.2 +0 P= RrQy 1000= 24x 0, Qy = 204 m/sec Hore, Procure drop = pressure drop in trunk roadway + pressure drop in spit A 4000 = 2.2 x (20.4)? +086 x Q%, Qn = 9.9 m¥/sec and Now, Or = Qu Qe 203299405 Qy = 105 m/sec 235 Amine fan cunning a 300 rpm delivers 150 m/s of ar ata pressure of 900 Pa Fan and motor efficiencies are 75% and 90% respectively. Ifthe fan speed is reduced to 250 pm, the saving in electric power Input to motor in kW is, — Answer : 82t0.86. Solution. PQ x 107k ‘Actual power developed (Pac) 19 Poe = 900 150 x 1079 = 135 AW ‘The relation between power (P} and fan speed (Nis given by- Pons was Gi Pacd2 = 78:12 kW output power Input power to the motor = oa ef ficiency Therefore, i 35— 7012 Sovin In elec power input tothe motor = SE— 7822 ay = 8027 kw 236 Subsidence profile function, log theaters cross selon over at gual pane tivenas (2) = 0.8[0.996~ tan (222). tere = dance (nom the ition pot nD = depth nf seam, Considering SS ——— agrees ora depth of250mis Answer: 2021. Solution. 20 Where, point of infection pont of zero subsidence s() = 0a[0996~ tann(®)),m ‘point a, 5G) = 05 then 0 =08[0996~ wah (22)] 0.0332 = tanh™*(0996) o.aazx = 4 in (+2) x=9356m ‘Therefore, Angle of draw (f) = tan"? (22) p= 208° 0.37 A goat void of 250 m" is filled in 3 hours by hydraulic sand stowing method, Density of the a sands 2.6 tonne/m3, I the fl i factor of goat vod is 0.9 and sand to water ration the stowing mixture is 1.0 tonne to 1.1 m®, the stowing rate in m? is. ‘Answer : 286 to 293, Solution . Volume of sand required = 22°*92 = 75m? hour ‘Mass flow rate of sand 5X 26 = 195 tonne /hour sand _ itonne Volume of water required = 195 > 1.1 2145 m?/hour Hence, Stowing rate = volume of (water + sand)required ,m? /hour = 7542145, m? /hour = 289.5 m* /hour 0.38 Asingle-acting reciprocating pump delivers 0.018 m?/s of water when running at 45 cycles per minute, The piston diameters 300 mm and stroke length is 400 mm. The volumetric | efficiency ofthe pump in % is Answer: 83 t0 87. Solution . Volumetric displacement Piston area x stroke length X03 X04 mt (028m? ‘Theoretical discharge = volumetric displacement x rpm a 939 a0 0,028 x 45 m? /min = 0,021 m/sec ohmeticeficency ne) = Aaelie x 100 = 82s 100 86% Match the method of mining with strength of ore body, type of support and ore body eometry strength support Geometry Method ». Strong unsupported X.Tabularand steep ——1:Cut and fill Q.Moderate —-M. Artificially supported Y.Tabularandfiat 2 lock caving Rowesk 1 Self eupporting Z.Massive and steep 3. Roomard pillar (8) P-MX3, N22, R (0) PAX, N23, RMYZ (0) PENS, OMA, RAZZ (0) P24, NYS, RMXZ Answer (C) ‘Armine air samples contains Cif, CO, y,N,and 0. The mine air analysis using Haldane apparatus gives the following results expressed in percentage of total sample volume, Total contraction after combustion 110.0 C0; formed after combustion 160 9s 02 consumed in combustion “The percentage of CH in the sample analysed is B Answer :3.8 to, Solution. Hydrogen, methane id carbon dioxide react with oxygen as follows = 2, +02 = 24,0. waft) CHy +20; = 60, + 2810 2 260 +0; = 2605... 13) From equation (2), (2) & (3, the folowing relations are obtains Contin = 3+ 2040 nee) (Carbon dloxide = CH, +60 Orygen consumed = 21h + 264 +00 46) From equation (4), (5) & (6) +z = Contraction ~ oxygen consumed City = BEcotratin = 04 My = 10-95 = 05% CH, = 2495 242% 2.41 The ital investment fora small scate mining projects Rs. 5.0 crore, Annual cash inflow fora life period of 4 years is given below, Year Cash inflow (Re crore) a 45 2 20 a 20 4 45 24 “The net present value ofthe project a an annual ciscount rae of 10% in Rs crore fs ———— Answor 105 t0 0.6. Solution . We have Net presentvalue = K+ CBS + cB te tae NV=—S+ayon t iaane Grae Gray NPV =0.8 Crore rupees. 242 Given the following near programming problem, Maximize = 34, +482 subjectto xt 6 Bey 3a 39 xy 20x20 ‘The commer point feasible solution in terms of (42) 18 Was) (8) (225,15) (105,10) (0) 225,15) Answer (0) sol 25 ‘The rms torque forthe motor in kN-m is —____ Answer: 106 and 133, Solution . 26 Torque (ktm) ‘Time (see) fest feiaear rms torque, Mrrn ae where, P= ty fp + ty = running time and ¢ = rest time fpoo® x 10-4 120% x50 + 208 «20 tems torque, Mrs = [eee 204 12 30 22 bette 40430420430 Myms = 109.55 kiN.2 1.44 Altborne PByp concentration in a residential area is monitored for 24 hours bya respirable dust sample, Initial and final weights ofthe flter paper are 2.3425 g and 2.6996 g respectively. The average arlow rate during samplings 3.2m /min. The PM 19 concentration ofthe area in pgm” {s ——— Answer :220 and 228. Solution. ‘Total suspended sollds/suspended particulate matters given by- a T55/5PM = 28 gf? Where, ‘A- weight of fiter plus retained solids in 6996 gm 8 - weight of clean fiterin g =2.3125 @m (C-volume of sample filtered in m™ Given that C= 1.2m°/minute 24 hows = 24x 1.4.x 60m? hereto, LL —e TS5/SPM= 224 yg/ m? 45 The assignment problem given requires four diferent jobs to.be done on four diferent machines, = Machine Je an _[ [Ms [ie hk a [as [ae | 20 eS res hb 0 | a6 | a | a i Ie a | [as | as The minimum cost of asignmentis —____ ‘Answer 116, Solution , Step 01. subtract the smallest element of each ow from all the element ofthe row. $0 there Will be atleast one zero in each row. 28 tote te ls sep 02. Subtract the sim element ofthe column not containing ero from al the elements of that column, step 03, Draws minim number fines to cover al the ero in the abe number of fines drawn equal to the order ofthe mates, then optimum souton f obtained. na, then sablrac the smallest uncovered element om athe uncovered elements add ito the ‘elements that les atthe intersection of ines SteP Of romine rows sucesiely until row with exactly one unmarked eos found, Mar (C3) this zero, Mark (X) all other zeros inthe same columa Thisis Feasible solution, Hence, Minimum eost of assignment 27436424429 16 0-45 Acceleration ofa particle moving in astsightline is expressed by =2 Wheres denotes distance (mand t, time (3). At time t0, the dstance and velocity ofthe Paricle re Om and 3 m/s respectively The distance travelled by the particle in m after 3sis (apa 6 (9 (ois Answer (0) Solution or 30 Now, fdv = feat Ve tE HE cement) ceuation (2 becomes, a. _ (243d 1 BEEK enn +0+k K=0 Equation (3) becomes, safest 2.47 Rockbolthave length L= (150+X) em, where Xis a random variable with probability density function a J@)= F0~38,f-20282 ©, otherwise '¥ 95% bolt length () fein the interval 250-¢ to 150 + com, the value of cis. Answer: 1.88 to 1.92, Solution . Probability density function of random variable Xis defined by- [Croan 2-48 The properties for a bivariate distibution of two random variables X and ¥ are given below. F(X) =24, BW) = 36, £(X2) = 702, B(¥2) = 1524, E(xY) 004 ‘The correlation coefficient between X and ¥ is Answer : 0.8 and 0.85, Solution ‘The correlation coefficient ofthe bivariate distributed random variable X and Yi Corrx,yy = S22) veo) Whee, 2 ou (Ky) = covariance between bvarate dstributed random variables X andY corr (4) BY) — Hay £{X) = uy = mean value of X mean value of, and eWay variance of X= 02, = EX?) — [E@OP 7) - (ECO Variance of ¥ ‘Therefore, From equation (1) st Core 6M) = Fee VET OE coun xu = pele Corr (KY) = 083 0.49 biaxial stresses at a point inside a pla are shown nthe gure 6 —* 2 8 All stress values are in Mpa “The magnitude of the maximum shear tres in MPa and its direction withthe x-axisin degrees atthe some point respectively are (a) 8.25,37.98 oy749,37.98 —(€)825,52.02, (0) 749,52.02 Answer (A) 3 Solution . Here, ay, = 10 MPa ty = 6 MPa ty = 2MPa Principal stresses - considering; > ay 13 = (5) Bee oy) Hy 129) 4 fCio- gene Oy = 24825 6, = 6.25 Mpa taking 4ve ) and 0 = 10 MPa (+ taking ve) | Mani hearse gay) = 2252 = 8.25 MPa, and Direction of shear stress with x-axis is given bys= 50 Acer tunel is constructed ina bavi far eld stress veteal stress py and hoionta tess Kyo) as shown inthe figure. 4 » ithe rai ofthe tangent stress measured a the boundary paints Aan Bs 3 the value of oo Answer: 0.6. Solution . ‘Tangential steess(ve) is given by’ 35 carafe (04H (148) + 1090438) 0920} oneal But st boundary a= hen equation 1) becomes 004 = PUL) + 204-1) 082) rene) At point, equation 2} becomes putting = 0° (©00)4 = PLA K) $201 ~ 1) nena) ‘At point 8 equation (2) becomes, putting @ = Coady = PAK) = 201 = 2) case von) (00) 1045920) sea ~ i= Hem 20=HT k=06 Peak particle velocity (PP) at points A and 8 are measured fora blast pattern as shown in the figure, i ‘Ot: Blast holes In initiation row O02 2: blasthole second row after delay Delay: 25 me The relevant data are: Amount of explosive perholein the "row + S00kg ‘Amount of explosive perholein the 2" row + 475 kg PPV at point A, #18 mm/s PPV at point B +: 20mm/s Considering the following relationship, u(3) me notes the distance from the blast row to the measuring point and @ (inka), ppt Where (in m) de maximum charge per delay, The site constant Kand n respectively are (a) 1002,3.13 (0)622,292 (C) 823,259 (0) 1245, 2.99 Answer (C) Solution. We have, ppv =K(4) momises At point A, cnt) At point 8, 10 2) From equation (1) & (2) Putting n= 2.59, in equation (2), then 37 k=823 252 Copper ore of average grade 0.65% Is mined, milled smelted and then refined. The fo Information is available: Mi recovery rate Average grade in mill concentrate 120% Loss in smelting process 2 5 ke/tonne of concentrate Loss in tefining process 2kg/tonne of blister copper The amount of refined copper obtained per tonne of ore in kg is (4) sa0 (6)5.37 (ssa (0)6.50 Answer (8) Solution Grade of ore - 0.65 % Copper content pr tone of ore = 24 1000 = 65 ky S52 kg amount of copper per tonne of concentrate Amount of copper per tonne of concentrate ‘amount of copper per tonne of ore 5) Copper recovered by mil = 6.5 x ©) Concentration ratio Now, 100 kg of concentrate has 20 kg of copper 1000 kg of concentrate has = 200 kg of copper 4) smelting loss per tonne of ore = 5ke/tonne of concentrate = 5 4g/36.17 tonne of ore smelting fos 138 ke/tonne of ore Blister copper produced per tonne of ore = 5.52 — 0.13 38 kg 38 «) Refinery oe for 5.38 of ister opper = 2253 = 00107 ene, Refined copper produced per tonne of ore = 5.38 ~ 0.0107 a7 ke “The ratio of horizontal to vertical in-situ stresses, K, at a mine fild varies with depth, D ass (om as aM sas rine unit weight of overburden rocks 25 KN/m® the horizontal stressin Maat depth of 00 mis Answer 19.0 9nd 1925. Soon. Vertical stress (oy)=¥ 8 Where, _younit weight of rock hh=depth below earth surface 10000 ké/m?, and 9175 iow, relation between horizontal stress (vy) and vertical stress (a) is given by one kay 9175 x 10000 kN/m? 39 y= 19.2MPa 54 Acoal seam of 2m thickness is extracted by a longwall retreating panel with face length of 420m, Web depth of sheater i 0.6 m. Average manpower in the longwall face ina shift is 20. ‘The specific gravity of in-situ coal fs 2.4 Ifthe shearer makes 4 fulace cuts in 3 shifts, the face OMS in tonne is Answer: 13to.14, Solution = Production in one shift (tonne) OMS = numbers of workers in one shift In 3 shift = full face cut , then ‘m1 shift =4 full face cut = 1.34 full face cut roduction in one shift (tonne) OMS sumbers of workers in one shift 2x 120x06x14x134 Ooms = 2% 120206414134 OMS = 135 tonnes or = Production in three shifts (tonne) numbers of workers in three shifts coms 2x120x06x14x4 3x20 135 tonnes O55 Aleaded dumper of total mass 75 tonne, having wheel diameter 1250 mm, runs on a haul ‘oad which offers an average specific roling resistance of 260 N/tonne. The engine develops 0 amaxle torque of 15 kN, Tho starting acceleration of the dumper in m/s ‘Answer 0.058 and 0.065 Solution . We know, Torque = Tracive force xradius a5 kim = Tractve force x > “active force = 24 KV ‘The tractive free exertel by engine Is given by = “Teactive force = accelerating force +fetional resistance 75260 24 = accelerating force +5 a Accelerating force = 4.5 kN = 4500 N Now, Accelerating force = Mass xacceleration 4500 = 75000 x acceleration ‘celeration = 0.6m?/sec fas6 choose the appropriate word/hrase ou ofthe four options elven below, to complete he following sentence: ‘Apparent ifelessness dormant fe, {A) harbours (e)leadsto —(C)supports (0) affects ‘Answer (A) 0.57 Fillinthe blank with correc iiom/phrase. “That boy from the town was a __ inthe sleepy village. (a)dogout of herd (8) sheep from the heap a {C)fish out of water (0) bir from the flock Answer (C) 258 Choose the statement where undetined words used corey {A} When the teacher eludes to fferent authors, he is beng elusive, {®) when the thief keeps eluding the police, hes being eusve (hatter that are dificult to understand, identity or remember are allusve (P)wirages canbe alusive, but a beter way to express them silusory. Answer (2) 59 Tanyais older than Erie. tf older than Tanya, Ericis older than Clit, the first two statements are wue, then the third statement is (a) Tue (6) Fae (6) Uncertain (0) Data insufficient Answer (2) 16 Five cams have to completsin a league, wth ever eam plying evry other team exaety nee, before glng to the next ound, How many matches wl have tobe eld to complete the league round of matches? (4) 20 (020 oe (15 Answer (8) a Solution “the numberof all combinations of n tings, taken rat a time is oo Ne = Fail oe cE Se, =19 61 Select the appropriate option in place of underlined par of sentence: Increased producty necessary refets greater efforts male bythe emPlyees (a) increases in productivity necessary {8} Increase productivity is necessary {€) nerease in productivity necessarily {0) No improvernent required Answer (C) «62 Givenbelow are te statements followed by two cncsons. Assuming these statement 19 be true, decide wiich one logically follows. Statements: 11. No manager is eader. 2 alileaders are executives Conclusions : 1L.No manager is an executive, 2. No executive isa manager. (a) only conclusion 1 follows. a (8) Only conclusion 2 follows. (€) Neither conclusion 2 nor2 follows. (0) Both conclusions 3 and 2 follows, Answer (C) 62 nthe Given gure angle is aright angle, PS:NS = 3:1, A:OT= 5:2 and PULUR of triangle QTS is 20 cm®, then the area of triangle PQR in cme? Is Answer : 280, R 7 Ir * 3 @ Solution. {In similar Wiangles APQR and Agr: Sow ro ao 4 Area of AQTS = 4x QT x QS 20= 2x OT x QS x woof) ‘Acca of APQR From equation (), 20 (¢xPe xn) 20 = 4 (areaof APR) Hence, ‘Area of APQR = 280 cm? 2.64 Right triangle POR isto be constructed inthe xy-plane so thatthe right angle tangle fs at P ‘and line PRis parallel to the x axis. The x and y coordinates of P,Q and R are to be integers ‘that satisfy the inequalities: 4 SxS Sand 6 < y < 16. How many different tangles ould be constructed with these properties? (w) n0 (093,300 (219900 (0) 10000 Answer () Soliton, No-tenes ' haben sts Ag+ Dechairs ( X, #X,) 45 ass Yolo chaies Kh *%) Total triangles = 10x 11 x 9x 10 = 9900 [Acoinis tossed thrice, LetX be the event that head occurs in each of the frst two tosses. Lot Ybe the event that a tail occurs on the third toss LetZ be the event that two tails occur in three tosses. Based on the above information, which one of the following statement is TRUE ? (A)Xand Yate not independent (8) Y and Z are dependent {C)¥andZare independent (0) XandZare independent Answer (8) 48 Gate Solution-2014 ecoetficient of stale friction 1 Ablockof weight 100 kw rests on a floor as shown inthe figure. Th 5, Aforce of 4 kN is applied horizontally on the block. between the blockand the floor f 0. ‘The state frictional force in kM is A00KN | coeftcieneot state retin = ae coca (ection #05 (ay225 (0) 50.0 1c)55.0 (0) 100.0 ‘Answer (8) Solution. rn Where , Fis fietional force Reaction force 20.5 = Coetficent of state felon Now IV=0 100-0 R= 100KN Hence, fretional force (f), F=nR a 05 «100-500 2 Aspring of constant stifinss ks stretched from point Ato point 6 displacement u in the igure) by a force F. The potential energy ofthe spring is exprassed by 9) 2h (0) Sout re 0 kur kus Answer (A) sotion, Potential Energy of springs given by = Kx2 2 Conservative force Conservative forces defined as one that does mechanical work ‘ndenendent of path of motion and such that the works reversible or recoverable Therefore incase of spring, the mechanical work oa conservative force i considered tobe loss ‘in potential energy of spring and tha is:= u W = Fu Hence, potential Energy (2) of springs given by Q.3 Gs isthe induced stress and GF is the nsitu stress at a point below ground, the ‘stress concentration’ at that point is wf of de Answer (0) Q.4 The components of state of stress ata point in x-y plane are given as Ox =5 MPa, Oyy= 10MPa and ‘xy =~ 2MPa. The sum of the principal stresses acting on the x-y plane in MPa is, Answer: 15 t0 3. Solution, Given that o4,.=5MPa o,=10MPa ty =72MPa Major principal stress given by: . soil -0,F os} om hsnioye{ Hs 10) +¢ av} 0.7015 MPa, ‘and minor principal stressis given by 49 5+10)-{265-107'+¢29 @,= 430 MPa Hence, Sum of principal stresses Is glven by Yo=oto = 1070+ 430 =15MPa QS Theangle 15°25” Is expressed in hours, minutes, and seconds as (ay 1201.67" (8) 1'20""16.00" (cyohzaina. 7mm (0) of21™16.00" Answer (C) Solution. We know, 1 50 and 01.67 Hence, Angle 5°15'25" = 0*21™1,67° 6 —Acircular curve has a radius of 200 m and deflection angle of 65°. The length of the curve in mis (ayaa (0)227 (0235 {0) 262 ‘Answer (8) Solution. We know, Or Length of curve = Angle x radius 65° x 200, 27 m Q.7 The weight strength of ANFO of specific gravity 0.8 Is 912 kcal/kg. The weight strength of an ‘emulsion explosive of specific gravity 1.2 is 850 keal/ke. Bulk strength ofthe emulsion explosive relative to ANFO in percentage is ‘Answer: 135 to 142. Solution, si strength = tative welpht strenuth of exposive x56. of explosive Fete trent ative welghestrogtho] ANE Sao aso x12 So, Bulk strength of emulsion explosive = 252% 12 2 ae e Siz x08 40 = 140% 28 Inacut-and-fill ope, the main purpose of back filing is to {A) reduce ore dilution (6) prevent high stress concentrations in far field domain (C) prevent displacement due to dil (0) improve ore rehandiing Answer (¢) 9 bypass valve in a compressed oxygen type self-contained breathing apparatus fs meant to (a) release accumulated nitrogen inthe breathing bag (0) release excess pressure inthe breathing bag (€) upp oxygen directly to wearer in case pressure reducing valve does not function (0) flush out the apparatus with oxygen on opening the cylinder valve Answer (C) 10 Given Sis the setting load and Vis the yield load ofa hydraulic prop, the correct relation (aysey ()s>¥ (osz¥ (o)s=¥? Answer (A) 2 key follows exponential decay (where kis 2 Solution of the differential equation dy/t conan) fr x 10218 (A) ko {B) k<0 mk {o) ‘Answer (B) ay rs et0 2 sonar eo son Given vectors a=2i-3) beKis aj when two vectors are orthogonal to each other then, ab-0 (21-3). (Way =0 2K-12=0 43 Which one ofthe following i the mast kely mode of slope aiue for waste dump {A) circular (8) wedge (©)Plane (0) Toppling Answer (A) G44 The occurence of headin single toss ofan unblased coins given by a random variable x The variance of Xis Answer :0.25 100.25, Solution, The occurrence of head in single toss of an unbiased coi So, random variab 12 Hence, variance of random variable x = ( ‘) 2. et 4 Q45 —_Thedlivergence of the vector Germ eyienpis (A) vox (8) x-y Answer (8) Solution vec eyed) Gwen vectors, +kK2\generen 2) ave(1Z4 2x2) Ly sy? ahh sure 2fyasth 2heon) div. Fanyex div. Fex-y aa Theva otlimese!! (ay-a (eo qa (0) non-existent Answer (0). Solution. x1 Given function, £(x)= lim © 6 x90 X Putting x= O+h, when x» Othen h->0 ll BS o+k Rf O+ 55 Hence, RF (04h) + Lf (O-N) ‘Therefore given function isnot existent. For Indian coal mines, the ‘maximum allowable concentration’ of respirable dust containing 7.5% free silica in mg/m? is (w20 (22 (ors (oy27 Answer (A) We know ie Ue yin’. Arestold lies 4 epwableatica "©!" “Bread ysmeim = 2mg /m?> 56 ass aaa 220 aa Given kis the thermal conductivity, pis density and cis specific heat of a rock sample, the thermal difusivity of the rock sample is we we o£ ms ‘Answer (0) Cyclone, bag filter and scrubber can be used for control of (A) water pollution (8) ar pollution (C)soil pollution (0) noise pollution Answer (8) ‘Armine waste dump of pH5.2 can be neutralized by adding (A) urea {8) calcium carbonate (C) sulphuric acid (0) sodium chloride Answer (8) A flat coal seam of thickness 1) 3 m is excavated and broken root rack has completely filed the space created dus to extraction as shown in the figure. If the bulking factor of roof rock is 1.2, the caving height (H) in mis, Answer: 30 t0:30. 37 Solution, Given that, Bulking Factor of rock = 7s of rack before Blasting Total area =x e+ 2x wc Now, (wes ew cn) x4 az # of coal sample weighs 10 kg in air and 2 kg when immersed in water, The specific 58 gravity of the coal sample is, Answer 1.25 to 125. Solution. Specite gravity = 7g where, wy weight of solid in air =10 ke Wa weight of solid in water» 2 ke Hence, specific gravity of coal sample 23 tna borehole log of 1.2 min length, recovery of rack cores in em seven below 20, 8, 15, 8, & 4 3, 9 10,4, 5, 10 ‘The ROD in percentage is (0318 (0458 (0)50.0 (ay2s2 Answer (C) Solution. weknov, ength tof core pleces 210. tenath yg Total length of core rum = aD ga « (20425220480 99 = 024045 +03 408) m O24 0as HOSS ODT, 309 ne am 59 024 Anunderground coal mine panel produces 520 tonnes per day ‘deploying 220, 200 and 192 Persons in three shifts As er CMR 1957, the minimum quantity of arin m/min to be ‘delivered atthe last ventilation connection of the panel i. Answer :1320 t0 1320, Solution, Production per day = 520 tonnes Total employee per day = 220 + 200+ 192 =612 Here, ‘Total employee per day > Production per day 50, Minimum quantity of arto be delivered shall be according to employees per day. Hence, Minimum quantity of arin mS /min tobe detvered atthe lst vnttation connection of the Panels = number of workers in largest shift x6, m?/min 20x6 = 1320 m/min 0.25 In PERT network, the activites onthe ertcal path are a band c. The standard ‘evitions ofthe durations of these activities are 2,2 and 1 respectively. The variance of the project duration is (3 (8s 9 (oys2 answer (C) Solution, Given, o=2, In PERT network, (wariance)ane = (variance), + (varlance), + (variance). (wariance)aye = (@?)a + (Ov + 67% (wariance)ape = 22+ 22 +17 = 9 0.26 Apart Pisin equilibrium as shown in the figure. The magnitude in KN and the orientation 0 in degrees ofthe force F respectively are y Pe a x 150kN 50" 200 kN (ays, 16.3 (o)221.2,232 — (c)1023,53.4 (0) 1803, 739 ‘Answer (0) Solution ot Fcoso 200c0330° Now, ZH =0 = 150 + Fcos0 + 200 sina0”= 0 Fos 50 And, 2V = 0 F sio0~ 200 eusso?'= 0 F sind = 173.2050 ... From equation (i) & (i), iis found that, 173.2050 50 ‘and tand= 4641 tan“ (3.4641) 3.9" From equation (i), F cos® = 50 Fcos73.9"= 50 2 50 0373.9 = 180.3KN 27 [Adistributed load of 4 kN/m acts on a beam of 6 m length supported by a hinge anda roller as shown in the figure. The distance in m of the point of zero shear in the beam from the point Ais__- Answer :3.4 1035. Solution. anyon Now, DY =0 6 1 Rut Re ZX Ox4 RH Ry = TZ KN oennenenell) ‘Taking moment about A, Rex Bx oxcax nyo Now, (S.F), or (S.F). pox toc x But (S.F).=0 22 ‘Ady rock sample of diameter 0 mm and length 100 mim weighs 300g. After saturating 64 creased to 330 g. The porosity of the in brine solution of specific gravity 1.05, its weight in rock sample in percentage is Answer :14.25 to 14.65. Solution. volume of vlaftouia Porosity © oral volume of samp otal volume of sample = 721 = 3.14 x (25)* x 100 396250 mm _ mass of ta “pociicaravly of ul eens OF wate (10-3009 Tos xrg00 kam sega ‘Tos 100080 Tas xi0003000 or s0x1000 mm” 499, Hence, Poroslty = 354956 1.08 xA000%1000 rem 21455% 4429 Aint plane of length Land dip 6 intersects the tov of 2 pe 2 shown in the figure. The weight of the shaded block is W. Unifor Internal fition ofthe joint surface are 2 mater pressure Pacts normal to the joint planes If the cohesion and angle of snd respectively, then the expression fo ‘safety factor’ of the shaded blocks 65 ty Etats Let cor6-LP tang Owain ‘Answer (¢) solution. Foto aft = tig re ert free From Mohr Coulomb failure criteria, Shear strength = r= c+ otane {e~ cohesion, oy —normal stress, ~ Angle of internal fr ‘ction) 66 therefore, Shear force = rA = (c+ dq tand).A= (6+ N/Atan DA So, as per elven condition : Resisting force = shear force Le+ (Wacos® - PL tanb And driving force = w.sind et (w cox —Leytand Hence, Factor ofsafet oor 20 Thelengths and standard eros of thee sections A, BE, and ofa straightline AD are Bg rene Oh ec aa 0.025 m. “the standard error in total Fength AO in is (a) 20.0836 (6) 10.0350 (q)30.0250 (0) 10.09 answer (A) Solution, egy = $0021 eye = 0.029 ep = $0025 “The standard error in total length AD— be, = V@O2I + 0.029) + 0asy de, = 0.0036 or ¢, = + 0.0436 6 31 The bearing of sde AB of a regular hexagon ABCDEF is SS0'10E . the station Cis “osterly rom the station, the whole ele bearing ofthe side Bi (ayoss'25” (8) 6550725" ()69"15'25" (0) 69°s0'0" Answer (0) Solution, Hence, whole circle bearing of 8s 65°50°0", Note In hexagonal ABCDEF ERALB% £0 20> fe = Le = 3298 052 Ina roomand-plliestope, bench blasting is conducted Using ANFO having density of 800 Power factor of 0.48 ka/tonne, the spacing and burden ofthe blast pattern in m respectively are 6 (c)52,40 (013, (9)2.0,2.6 (8)23,11 Answer (8) Solution, Given that ANFO density = 800 kg /m> Specific gravity of rock = 2.5 Hole dia. = 100 mm spacing) urden) = 3 Power factor = 0.48 kg/tone, and charge length of hole = 80% of hole length Amount of expiosive (k9) Powder factor = Figsted material Comme) 0020 (005) ohare tenath 048 Sicehotelength es og = ine au tte moo xa14 (005) 09 0.48 = Sapa 5004324 x 005)" x08 a1 x25 B=18m ‘and spacing 69 (2.33 Match the following for ore handling operations in an underground metal mine Arrangement Description (©) Drawpoint (0) arrangement that prevents oversized rock to pass (2) Ore pass (i)a system of vertical or near vertical openings for transferring ore from a stope toa single delivery point (Ry Griaty (wi) place where ore can be loaded and removed (5) Finger raise (WV) a vertical or inclined opening used for transfering (aypav,a. (8) au, aay ra, (€1Ps1, cay, ra, san (0).,.04, na, sav Answer (8) 2.34 The following characteristic curves (P,Q, RS) pertain to rotary driling in rock Title of the curve 1 Torque versus RPM Rate of penetration versus uniaxial compressive strength of rock Rate of penetration versus weight on bit |W: Specific energy versus weight on bit Match the curves with their tiles (4) Pat, aay, Rays (©) Paty, ra, san (opay, act Ra sat (0) P4, as, Ra, sav 70 Answer (C) 35 Thehoight H of drawpcint in a sublevel caving stope 39m Ifthe angle of repose (o) of sroken ore is 38°, the digging depth yof the loader as shown inthe igure" a Answer: 2.7010 2.75. ore ene rath Lerrnynieirteerrere Solution. Given that, -y=35° n Now, In Sauc, tan 625° al B 617m Now, InAAgo tans O26 Foran explosives company, the probability of producing a defective detonators 0.02. the Probability that alot of 50 detonators produced by the company contains at most 2 dofectivedetonatorsis ‘Answer 0.90 t0'0.94, Solution, From Poison distribution, Probability of observing x events n given interval n The probability that at most 2 defective detonators is 937 Thearea enclosed by the curves y= and * for x € 10, 16 ayaa (a6 owe o. Answer (A) Solution. Curves interest at (0,0) and (1,1) and we see that x < 2? in this interval Therefore, area is given by 2.38 The value ofa for which the function below is continuous at x= is 1@)=@xtaxt, xo4 (4x43, x>a (ays (0 (os (0)10 Answer (¢) Selution, Function is continuous at x=1 MH Qxtax%ee r+ Decco m QxLeaxt? =4x143 2eanaea ieseries a+ ar tartar + went att + munfor I< Is {A} alter) (8) a(t-r) (0) af(ten) {0) af(1-1) coer} ah wen [el re | ‘Since, numerical value of ris less than 1. When value of n increases, then value of r™ decreases. When n tends to. then value of 1” tends to 0. So sum of infinites series will be rm. if the pump efficiency is 75%, the input power to the pump in KW is (a)a7.20 (8)49.05 (1333 (0/750 Answer (A) Solution. Given that, Discharge rate = 2000 L/min Total head = 200m Pump efficiency = 75% pgxt0 Pimp erritenay ®™ Input power to the pum "ump pressure = pgh = 1000 x 9,81 x 200 = 1962000 Pa 1000 I/min 2000010" 3/500 Now, Input power (Pw pump = 952000 200028°7 0-7 G41 Adagline is required to remove 3,00,000 m*of rock per month on the bank volume basis. Consider the following data for the dragline operation. Effective working hours per month = 450 Bucket fil factor = 0.8 cycle time = 65s ‘Swell factor ofthe rock =1.25, ‘The minimum bucket capacity ofthe dragline in m?is (7.70 (6)9.63 (12.04 (0)18.80 Answer (0) 8 Solution, Given that, Dragline productivity per mont effective working hour per mont ‘Bucket fill factor = 0.8 ‘cycle time = 65 seconds Swell factor of rock = 1.25 dragine produetvty per hour = 2 66.6710 Now, dragline productivity nm? / isgiven by eons ctor bucket enpaciy yee tine Xswel Foetor sooxpanducket copactey 666.67 Bucket capacity = Se Bucket capacity = 18.80 az Adirectrope haulage pulls tubs loaded with coal through an nine of length 500 m having jonal data. aninclination of in 6. Consider the following ad Capacity of tub=1.0tonne, 500 kg km per hour, Coettcient of friction between whee! and ral = 1/60 n Coefficient of friction between rope and érum=1/10, ‘Mass of rope per meter = 1.5 ke. The minimum power required to haul the tubs in kW is (9) 345.50 (2) 348.60 (350.10 Solution, (0) 365.50 Cal fo— vee tngine Tractve force required = G-+g 4 +F+ Fy G= Gravitational resistance of full tubes = (Weight of gross ioad per set) x. x inclination al G=1500x8%9.81x = « 196206 81 = Gradient Resistance of rope ‘ope length x rope mass per ke x x inclination 1 81 "50x15 n981y & =372625N F = Friction of full tubs 78 = (Welghtof dross load per set xg x coetfident of fition between wee! snd 1 Fa1500%9.61x {= 1962 fiction of rope ope length x rope mass per xg x coefficient of fction between rope and drum 1 Fy =800x15x9.81 x = 735.75 i 0 Therefore, “rractive force to overcome = 19620 + 1226.25 + 1962+ 735.75 = 23580, 5 m/sec Hauling speed = 9 kr inimmum power (P) required to haul the tubs = Tractive force x speed 23544x2.5 = 58860 W P=s8asKw as Acoal mine receives two bids for purchase of anew dragtine, The fst id quotes Ss 150 price tobe palin ful on delivery. The second bid quotes Rs, 180 crore 52 Price very. if the discount rate is 12%, the difference payable at the end of the third year after deli in uv between the first and second bids in crore of rupees is, = Answer : 21.80 10 21.95. Solution. NPVisgiven by where, The time of cashflow = Discount rate Ry = the net cash flow For first bids — $20, Re150Core, 15 12% <0.12 wore Here, (NPV); 150 crore for second bids 180 Crore, Here, (VPI), = Me = Therefore, ‘The dference in NPV between the first and second bids in Crore of rupees is 150~ 128.2 Q.44 Match the following. the context of underground mine environment Instrument Measuring parameter . Haldane apparatus 1 Humidity © Godbert- Greenwald apparatus. Airvelocity R. Hygrometer MM Mine ai composition S. Anemometer |. Ignition point temperature 80 (a) Pa, 4, Ral, SV (0) Pam aay, RA, Sat (€) Paty, ae, RS {0) Pt, Ql, RAV, SA) answer (8) 45 Amine airway having cross-section of 2.2 m x2.2 m and length 500 m contains 9 bend: Given that the airway fition factor i 0.01 Ns", shock loss factor for the bend is 0.07 and density of air I 1.2 ke/m, the equivalent length of the airway in mis ‘Answer: 502 to 503. solution, We have, Pressure drop in mine airways: _ when this equation is applied for bend then, becomes, ap, = M05 where AP5= Shock oss Ky = Friction factor for bend Sp surface area of bend ‘v= Velocity of air through bend ‘A= Cross seasonal area of end but APS=X.Py x-»Shock actor, Py~Velcity pressure at Hence, Kase? Py = MS pt = ae Putting allvalues then, 007 3x 12 = SAEE2*229*41 Ie, length of bend] 4 (y=231m Equivalent length of airways = $00 + 2.31 = §02,31m 0.46 order to estimate the NVP in a mine, measurements are made at the main fan as shown below, Fan speed (RPM) Fan driftpressure(P=) Fan quantity (m/s) ‘800 655 a2 950 730 355 TheNvPinPais Answer :258 to 263, Solution, We have Pepa? ‘When, fan speed = 800 rpm, fan drift pressure = 655 Pa Fan quanti 22 m9 /S 2 N+ 655= Ry x(82.2) “eo Where, N ve when, Fanspee 150 ppm, Fan dit pressure Fan quantity +855 m° /S N#730= Rp x(85.5)* N+730 _ 7 «i In both, conditions resistance of mine wil be same. Hence, Ry = Re ‘On solving, its found that, 260.42 Pa 0.07 Theresstances of two splits A and B are 0.35 Ns'm*and 0.05 Ns"m'* respectively. The combined resistance of the shafts and trunk airways is 0.4 Ns'm*. A booster fans planned to be installed in split to increase the quantity flowing through it, Assuring that the surface fan continues o operate at a constant pressure of 1000 Pa, the critical pressure of ‘the booster fan in Pa is ‘Answer :874 %0 876. Solution, Given that, Ba Resistance of split A= 0.35 Ns%m® Resistance of split 8 = 0,05 Nm Combined resistance of shaft and trunk airways = 0.4NS2rn. Surface fan pressure 1000 Pa ‘Now, critical pressure of booster fon is given by Where Py ~surface fan pressure Rg Resistance of splitin which booster fan installed Re- Resistance of shaft and trunk airways p= leone 2. =875Pa 2.48 Apitot tube is inserted in 2 ventilation duct withthe nose facing the ar flow. A vertical U- ‘ube manometer filled with alcoho! (specitic gravity 0.8) has been used for pressure ‘measurements such that 10.2 mm is read as the total pressure and 8,8 mm as the static Pressure, Given the density of air to be 1.2 ke/m’, the air velocity atthe nose ofthe pilot tube in m/sis We know, a4 Presse Pa) p= Bensiy oF fui kg/m? ‘g—Acceleration due to gravity (m/sec?) hh=Head (height) {m) Tota pressure =0.8 x 1000 x9. 282 pa =80Pa Static pressure = 0.8 X 1000 X9.81 x22; Pa = 69a ‘Total pressure = Static pressure + dynamic pressure Dynamic pressure = 80~ 69, Pa =11Pa Dynamic pressure where, p= density of air V—Velocity of air Baa x axe 12 v24.28 m/sec or a49 as {An illumination source $ shown in the figure emits light equally inal directions. Ata point A on the floor, the illuminance fs 5.0 lux. The ik ance at point B onthe floor Inluxis Answer :1.0t01.2. Solution, Given figure 6521.63" Now, @=tan-# Hence 8 180" - 0-90? = 180" 36° 2'11.63" 90° = 53°7'48,37" a6 50 Solution. Wehave, iumination of surface (Meter candle) = (Candle of source) x Se Here, Candle of source = 5x33 =45 lumens cos 04 =€0353°7'48,37" = 0.6 Hence, ilumination at point 8 wil be- ‘Two machines A and B while operating simultaneously produce a sound pressure level of {85 dBA ata point. When the machine A stops, the sound pressure evel at that point ‘edlices to 80 dBA, The sound pressure level at the same polnt due to machine A ‘operating alone in dBA is, (ay70.0 (8) 75.0 (800 (oys3.3 ‘Answer (0) We know, Leg = WO logso[Ef=g te - 10°19] 2 A) Olog yo] 1x10 10 411010 as 85» log 19 [108 4104/1 10° 108 4104/10 a or 10 =10" 104" in(to®—108} A in(00) 3.3.68 0.51 Awaste water effluent has BOD; of 80 mg/L and the reaction rate constant is 0.16 per day. The ultimate BOD in me/Lis (ays (8) 100 (c)120 (0) 245 Answer (D) ee 200-800, xf-e"*') where, BOD, 30D at any time t, mg/L = 80 mg/L BOD, = Ultimate 80D, mg/L. K= constant, reaction rate = 0.16 t=Time, day=5 day Hence, elven BOD = 80 mg/l, k=0.16,t=5 0 = 80D x(\-«-5*:6) BOD, = = 52 series of tri-axial compression tests conducted on sandstone samples reveal the following relationship between major and minor principal stresses a8 61=5043.0, [stresses arein MPa} “The cohesion in MPa and angle of internal friction in degrees of sandstone respectively are (a) 14.43, 30.0 (8) 14.43, 60.0 (c)0.21,739 (oyo21, 16.4 Answer (A) Solution. Given that and we have 8) oy = C+ tampa where, C, —Uniaxlal compressive strength land angle fis constant Comparing (i) an (i, we found that, C, =50MPa and tan = s4sine tan = ys suhere, Angle of internat fiction Ps 230% andatio we have, nodal compresive strength (o) = 222 0 = 26s 89 % Cx1443 Mpa 253 Skedetonator each having resistance of 15 ohm are connected in parallel. A uy ‘“rPloderis connected tothe detonators by two single-cre cables of resistance 3 ohm ‘each. The current inthe cecut in Ampere is Answer:2.2to24, Solution, Given that, Dy =D =D =D4=Ds =D6=150hm | i ‘and Cabley,= Cables, = 3 Ohm. Now, Total resistance =0.25 +3 + 250hm Hence current in the circuit. 28 =24 ampere 54 Thefallr and the repaerates of shovel ar 0.06 per hand 0.08 per respectvely. The availability of the shovel in percentage Is Answer: 40 to 40. Solution. Availablity = eat eima ~ Tpdine + down tine Where, MTF» Mean time to failure MTTR-» Mean time to repair Given that, Failure rate =006 hr~* Therefore , MITE = <1 -=1667hr cere O06) land Repair rate = 0.08 hr-* 1 Therefore, MTTR 25he 91 16.67 1667225 Hence shovel avaitabilty 0.40 40% Q55 © —_Theindividuat liability values of four sub-systems are given in the figure below. The «liability ofthe system is os as A 8 I p> c HH D oF tS Answer: 0,50%00.55. Solution. Given, Reliability values of sub systems A & B Ral 6x06=0.36 Reliability values of sub systems C& D Rep=05x0, Now, Reliability ofthe system is. =1~(1-0.36)x(1-0.25) =082 2.56 Choose the most appropriate word from the options given below to complete the following 2 sentence, ‘A person suffering from Alzheimer’s disease short-term memory loss (A) experienced {8)has experienced (C)isexperiencing (0) experiences Answer (D) 57 Choose the most appropriate word from the options given below to complete the following sentence. {s the key to theic happiness; they are satisfied with what they have, {A)Contentment (8) Ambition (0) Perseverance {0) Hunger Answer (A) 58 Whichof the following options isthe closest in meaning to the sentence below? “a5. worman, | have no country.” (A) Women have no country. {) Wornen are not citizens of any country. {€) Women’s solidarity knows no national boundaries. (0) Women ofall countries have equal legal rights. Answer (C) 59 many given year, the probability of an earthquake greater than Magnitude & occuring in ‘the Garhwal Himalayas s 0.08, The average time between successive occurrences of such earthquakes Is__ years. Answer 25 t0 25. Solution, Ey 4 p=00a= 4 100 for 1 earthquake Reverse probability earthquake 5 years ‘The population of a new city is 5 million and is growing at 20% annually. How many years 2.60 would it take to double at this growth rate? (8) 3-4 years (8) 4-5 years () 5-6years (0) 67 years Answer (A) Solution, We know, from compound interest S=Pa or Where, $~value of n periods P—Intial investments i= Annual interest ate n—Number of years Here, we can apply such that, 10 = 5(1 +02)" 94 ast sz tna group of four children, Som is younger to Rin. Shiv elder to Ansu. Ans Is youngest in the group. Which of the following statements is/are required to find the eldest child inthe sroup? statements 4. Shivis younger to Riaz. 2, Shivis elder to Som. (A) statement thy itself determines the eldest child, {6) Statement 2 by itself determines the eldest child. (c) statements and 2 are both required to determine the eldest child. {0) statements 1 and 2are not sufficient to determine the eldest child Answer (A) ‘Moving into 2 world of hig data wil require us to change our thinking about the merits of exactitude. To apply the conventional mindset of measurement tothe digital, connected World ofthe twenty-first century sto miss a crucial point, As mentioned earlier, the obsession with exactness Is an artefact ofthe nformation-deprived analog era, When data was sparse, every data point was critical, and thus great care was taken to avoldleting any point bias the analysis. From “BIG DATA” Viktor Mayer-Schonberger and Kenmeth Cukier 9 The main point ofthe paragraphs (0) The wentysfiestcomtrys a dal wort (0) Big data is obsessed with exactness (6) bracttudeis not etc n dealing with big data {0) Sparse data leads to a bas in the analysis ‘Answer (C) 63 The total exports and revenuies from the exports of a country are given in the two pie charts ‘below. The pie chart for exports shows the quantity of each item as a percentage of the total ‘quantity of exports. The pie chart for the revenues shows the percentage of the total werated through export of each item. The total quantity of exports of all the items ‘is Slakh tonnes and the total revenues are 250 crore rupees. What isthe rato ofthe revenue enerated through export of Item 1 per kilogram to the revenue generated through export of Item 4 per kilogram? Exports Revenues teem 1 wiz (o)24 (aaa (ayaa Answer (0) Solution, For tern, 12 Revenue= =~ x250 = 30Crore rupees 700" P u fxports =

x 5.00000 x 1000 = 55000000 . 100 bd Therefore, 30 feevenue generated through export of item, PerKlograms = 90 « a ‘sem’ 55000000 For tem v, Revenue = £4250 156 00 Pe 22. 5000000 «1000 = 110000000 ke 100 “Therefore, 15 Revenue generated through export of item V per kilogram = ——1> e ae per logrm = 79000000 Hence, 64 Xie 1 kmnortheast of. is 1 km southeast of. W is 1 km west of. Pis 1 km south of 'W. is km east of P, Whats the dstance between X and Qin km? (aye (oyv2 ow (2 7 Answer (C) Solution, 1 8x42, applying Pythagoras tneorem} We found that, x= 2 ‘Again in AXZQ [applying Pythagoras theorem} xa= (Za)? +z xa 12+(2P x0= 8 km Q.65 _10%of the population ina town is HIV*. A new diagnostic kit for HIV detection is available; this kit correctly identifies HIV" individuals 95% ofthe ti 1¢, and HIV Individuals 899% of 98 the time, A particular patient is tested nig this kitandis found to be positive, The probability thatthe individual is actually positive is. Answer : 0.48 to 0.4 Solution, Let total population = 160 ta + patent «100% 22. «x0 patient = 100% 7 = 90 atv patient = 100x 100 For patient to be Wve = 100.95 and to be-ve = 10x05 For patient to be -ve = 900.89 and to be +ve = YU xU.11 Hence, probability thatthe individual is 10x0.95, Actually postive= 55 oe, conOaT 48 99 Gate Solution-2013 Q1 Inthe Coward flammability diagram, the respective percentages of methane and oxygen at the nose limit are (a)142,0.0 (8) 24.,18,2 (c)5.8, 12.1 (0)5.0, 19.2 Answer (C) 2M the transpose of a matrix is equal to its inverse, then the matrixis (A) symmetric (8) orthogonal (C) skew symmetric _(0) singular Answer (8) Solution As pr sven condition aaa 2 AAR Hence , matrix is orthogonal. Q3 Inthe Moh’s scale of hardness, the minerals in increasing sequence of hardness are {A) calcite, gypsum, topaz, diamond (B) topaz, gypsum, calcite, diamond {C) caleite, gypsum, diamond, topaz (0) gypsum, calcite, topaz, diamond ‘Answer (D) 4 Aball of weight W is supported on smooth walls as shown in the following figure. Ry and R, are reactions from the walls 1 and 2, The free body diagram of the ball is represented by 100 ‘Answer (A) Solution . so as Solution . a6 a7 as For a 25 mm diameter spherical charge, the maximum allowable charge length in (a)15.0 (8)25.0 (930.0 (0) 150.0 ‘Answer (A) Explosive charges used in Vertical Crater Retreat method are normally spherical Spherical charges have been defined as having a length : diameter (1:0) ratio of 6:1, * diameter of spherical charges lenath of spherical charges = 6x25 =150 mm Long-hole drilling with crater blasting is used for the constru (A) winze (8) shart (C) raise (0) decline Answer (C) Rill stoping method isa form of (A) block caving (6) artificially supported stoping (6) underhand stoping (0) overhand stoping ‘Answer (D) ‘Transit theodolite is a (A) micro-optic theodolite (8) theodolite with face left and face right reading facilities, 102 (0) theodolite with stadia hairs {0) theodolite with two vertical circles ‘Answer (8) 9 Incubation period is NOT related to (A) crossing point temperature of coal (8) panel size {C) seam thickness (0) explosibility of coal dust Answer (D) ‘The rotational speed and cutting velocity of a drill are 350 rpm and 71.50 m/min ao respeetvely. The diameter of the rotary dl it in mm ie (ayes (o67 (oes (0)70 Answer (8) solution, Radial speed is given by: 2neor Where, ‘= rotational speed = 350 rpm radial distance 4 T1802 x 3.14 x 350 F 2 =0,0325 m =32.5 mm « diameter =2x 0.0325 mi 103 Q.11 The pressure on a phreatic surface is (A) less than atmospheric pressure (8) greater than atmospheric pressure {C) equal to atmospheric pressure (0) independent of atmospheric pressure Answer (C) Q.12 Events A and B are independent but NOT mutually exclusive. If the probabilities P(A) and P(6) are 0.5 and 0.4 respectively, then P(AUB) is. (aos (8)0.7 (cos (09 Answer (8) Solution . PlA)=05, P(e) = 0.4 + Events A & B NOT mutually exclusive, then Plane) +0 P(AUB] = P(A) + P(8)-P(AnB) ‘Or P(AUB) = P(A)+P(B)~P(A).P(B) PIAUB) = 0.5 +0.4-05 x04 = P(AUB) =0.7 (A) rotary diamond drilling {8) rotary roller drilling (0) percussive drilling (0) jet piercing Answer (C) Q.14 Identify the correct statement for a ‘normal distribution’. (A) Mean is greater than mode but less than median {8) Mean isles than mode but greater than median (©) Mean is greater than mode and median (0) Mean, median and mode are equal Answer (D) Q.15 An emulsion explosive of specific gravity 1.25 ls used for blasting in an iron ore formation having P-wave velocity of 3000 m/s and specific gravity of 3.20, For an explosive impedance to rock impedance ratio of 0.5, the desired velocity of detonation of the explosive in m/s is (ay aga0 (8) 4000 {€) 4200, (0) 7680 Answer (A) Solution . explosive impedance rock impedance =05 (oD), specific gravity of explosive _ (Wan), x specific gravity of ironore os (oye x125 105 = (Von, = ezseus328 = (VOD), = 3840 m/s ‘The number of ways in which the letters in the word MINING can be arrangedis (a)90 {8)180 {c)360 (0) 720 Answer (8) MINING = 6 N=2& + Required number of ways Under standard temperature and pressure conditions the theoretical maximum height in m to which water can be lifted using an airlift pump i (a) 10.33 (8)9.61, (017.45 {0)6.05 ‘Answer (A) Ina belt conveyor system, function of the snub pulley is to (A) clean the inner surface of the belt (8) clean the outer surface of the belt (€) increase the angle of contact of belt with drive drum (0) decrease the belt tension Answer (C) 106 .19 Inthe following figure, the coef... t of kinetic friction between the trolley and the surface is 0.08. When the block is released from rest, the acceleration of the trolley in m/s* becomes (a)9.65 (8)1.23 (c)1.09 (o)0.74 ‘Answer (0) Soluti = 107 Bv=0 R-80=0 R=80x981N Applying F = ma R= 80.0 1 - 0.04 x 80 x 9.81 = 80a ) | Ad) again applying F= ma, 10x 9.81 ~ =104 C) Solving equation (1) & (2) 0.74 m/sec 2.20 Twomeshing spur gear wheels of Module 6 have 24 and 42 teeth. The distance in ‘mm between the centres of the gear wheels is (4) 1000 (8) 198 (e126 (072 ‘Answer (8) Solution. ‘We have, Module = ameter 108 For first meshing spur gear whee! meters eS (diameter); = 6 x 24 = 144 mm Or (radius), = 72mm For second meshing spur gear wheel (diameter), 6 (diameter), = 6 42 Or (radius), = 126 mm Therefore, distance between the centers of gear wheel Q.21 _ Inanexperiment to study coal dust explosibility, itis found that at least 3.0 g of limestone dust should be added to a sample of 2.0 g of coal dust to ensure that propagation of flame does not take place. The explosibility factor of coal dust is {A) 60.00 (8) 20.00 (16.70 (0)3.50 Answer (0) Solution . Explosibilty factor is the ratio between inert and combustible content. Therefore, Explosibility facto 22 _A20msteel tape used in a mine survey is found to be 20 em short when compared with a standard tape. If the measured volume of a dump using the tape Is 4000 m°, its actual volume in m” is 109 ta)3881 (8) 3902 (c)3920 (0) 4221, ‘Answer (A) Solution. We have, ‘Actual volume incorrect volume x (2) Where, incorrect length of chai L= correct length of chain wow, actus volume = 00x 24)” maaan oe) 23 Amine worker inhales normal alr; whereas, the exhaled air contains 16.65% 0 and 3.83% CO2. The respiratory quotient of breathing for the worker is (a)o23 (8)0.89 (0.99 (0)1.3 ‘Answer (8) Solution Respiratory quotient = Sete C22. ams =0.89 2.24 Block economic values in Lakhs of Rupees for a se model are shown below. a fa dt fa jo ft - jo |o fo a0 At a permissible slope angle of 1:1, the optimum pit value of the section in Lakhs of Rupees is (ayo (ea 2 (0)3 ‘Answer (C) Solution ‘The following steps are used- (2) The cone fs floated from left to right along the top row of the blocks in the section, if there is @ positive block itis removed. {2} After traversing the first row the apex of the cone is moved to the second row, starting floats, from left to right stopping when it encounters the first positive blocks. Ifthe sum of al blocks falling within the cone is positive (or zero}, these blocks are removed .if the sum is negative the blocks are left and thee cone floats to the next positive blocks on this row. The summing, and mining or leaving process is repeated, a 110-1 110105241 = [ala {3} This loating cone process moving fram left to right and top to bottom of the section continues until no more blocks can be removed. (4) The profitability of the section is found by summing the value of the blocks removed. ‘The optimum pit value of section = +1 +1 = +2 = 2lakh of rupees am ‘opti pit tint 2.25 The boundary of a mine is plotted on a scale of 1:2000. fa planimeter measures the plotted area as 58 em the actual mine area in ms (a)s800 (8) 11600 (€)23200 (0)29000 Answer (C) Solution. mension of object ndrawing ‘actual divvension of ob ect Given , plotted area =58 cm? So, plotted length or width = ¥58 em 1 Siem * Boos ~ cma Tengen or with of wbject actual length or width of object = 2000 x ¥58 em So, actual area of object = 232000000 cm? =23200 m? 26 —_For a shrinkage stope the following data values are given Insitu tonnage : 9000 tonne Insitu grade : $.2¢/tonne Average grade of waste 1.4 g/tonne a2 Loss of ore in the stope Dilution Answer : 4.504.518. Solution. Tors Gakeigiony [Torre {gram Orein place 00 32 0052-46800 Diaton OH [= omoatem | 18 (ao) : Bn ‘Sreloss (108) | (-)Z xsoco4-Jo00 | 52 BOGE ET Tal 0 He ‘9900 il grade = AN g/m = 4.5 g/tone 27 _Inan experiment to determine specific gravity of a soll sample, the following data Is obtained: ‘Mass of empty pycnometer Mass of pycnometer with soil sample Mass of pycnometer with soil sample filled with water Mass of pycnometer filled with water ‘The specific gravity of the sample is Answer : 2.35 0.2.45 Solution . 3 20.4 g 516g : | BBG E 1 TO4g Where, Wy= weight of solid in air = 51.6~20.4=31.2 Ww, weight of solid in water = 88.6~70.4 = 18.2 ¢ «Specific gravity of sample ‘A cylindrical rock specimen of diameter 54 mm has Young’s modulus of 68.97 GPa and Poisson's ratio of 0.35. The rock specimen fails in uniaxial compression ata lateral strain of 0.01%. The axial load at ire In KN Is Answer 144.8 t0 45.5 taterat strain Tmgituainat stroma 0.01% _ 10-4 : - PBA Ifrom Hook's law, ¢ = P/AE ] 0.35 22 x 3.14 x Z7? x 10° X 68.9 X 10? 15.107 KN ‘An open belt drive connects two pulleys on parallel shafts that are 3.6 m apart as, shown in the figure. The diameters of the pulleys are 2.4 m and 1.6 m. The angle of contact on the smaller pulley in degrees Answer :166 to 169 eh aa Solution . sing = 22 Where, adius of larger pulley 1%, = radius of smaller pulley D = distance between centres of both pulley = 3.6m sing = 2-28 = 9.41 0-63" Angle of contact for smaller pulley = 180° ~ 2.x = 180° 26.3" = 167°24'0" 2.30 Atwo tonne mine car is released from the top of an incline at a height of 3 mas shown in the figure. The mine car travels 45 m along the inclined track and another 85 m atong the horizontal track before coming to rest. 850m ———— ‘The specific rolling resistance of the car in N/tonne us Answer : 225 to 231. Solution . Here, Potential energy=(rolling resistance from A to B)x(distance from A to 8)+ (rolling resistance from B to C)x(distance from B to C) mgh= pomgcos3.8° x 45 + jg x 85 19620 x 3 = pmg[cos3.8° x 45 + 85] 39620 x3 OF Rolling resistance = umg = os Rolling resistance = 453,11 N/2 tonne Rolling resistance = 226.6 N/tonne 31 surface miner with 2.0 m cutting drum width excavates coat in windrawing 16 ‘mode from a bench with effective face length 200 m. The cutting speed of the surface miner is 10 m/min and the cutting depth 25 cm. The density of coal is 1.4 tonne/m’. Ifthe average turning time of the machine at the face end is 5 min, the rate of production in tonne/hour becomes ‘Answer : 335 to 337. Solution. speed __ face tenth speed = Time 045 Now, total time taken in one cut X 200 x 0,25 100m? production by shovel 00x1.4= 140 tonne Now In 25 minutes = 140 tonnes a 36 tonne /hour Hence, rate of production in tonne/hout 32 Acore sample of a rock, having diameter 54 mm and length 108 mm, is subjected to axial loading. Ifthe axial strain and Poisson’s ratio are 2000 x 10° and 0.28 respectively, the value of volumetric strain, represented in micro-strain is, ‘Answer : 0.000875 to 0.000885. Solution. (1-20) a7 Where, € = axial strain = 2000x 10° 28 9 = poisson ratio 2.Volumettic strain = 2000 x 10-*(1 ~ 2 x 0.28) + Volumetric strain = 0.00088 33 flat bauxite deposit has thickness of 10 m with an average density of 2200 kg/mm3, The grade values and the sample coordinates are as shown in the table, ‘To carry out reserve estimation using triangular method, the triangles are constructed as shown in the figure. Sample No tials ‘Auina’s s[alslo[e coordinate, [0 [200 | 0 | 200 | soo yioodnatem [30 [mofo [0 [0 i ‘The alumina content in million tonnes, in the region comprising the three triangles Is Answer: 0.9t01. Solution. us (0.300) 2, 200,300) (0,0) 4 (200.0) 5 (500,0) 35440 +39 9s, = 3 = 38% 3 47440439 ro 12% 2 In,= 43% and Ay = 3200 x 300 = 30,000 m2 a = 3% 200 x 300 = 30,000 m2 hy = $x 300 x 300 = 45,000 m? Aigay 4a da, A Osa Grade of deposit @) 9 = 41.29% Volume of deposit (Vi= (Ay + Az ++ A) x thickness = (30000 + 30000 + 45000) x10 41050000 m* Tonnage of deposit 2200 x 1050000 tonne 19 10000 tonne = 2.31 million tonne “alumina content = 2.31% 0.4129 = 0.95 million tonne 0.34 Ifthe following linear system of equations has non-trivial solutions petytz=0 2xty-22=0 x42y-32=0 the value of pis (aya (0 a (7 Answer (0) Solution. Incase of Non-trival solution a py a 1 -2|=0 a 2-3 1-2) 42-3, 4)2 1 a nal-th oleh l= “P+At3=0 035 A bucket wheel excavator with 20 buckets of capacity 0.5 m° each, rotates at 5 rev/min. The bucket fill factor is 80%, The excavator loads on to 1200 mm wide belt conveyor. The cross-section area (m’) of the material on the belt is 0.18%, where B Is the belt width in m. The minimum speed of the belt in m/s 320 to avoid spillage of material is (y723 {8)5.79 (c)4.63, (0)3.70 ‘Answer (C) Solution. Productivity of bucket wheel excavator Is glven by: Q = 60x wheel rpm x No.of bucket x bucket capacity,m?/h Q= 60x 5 x 20.0.5 = 3000 m3/h = 0.84 m/sec Q = 0.84 x 0.80 = 7 m3 /see ‘And productivity of belt conveyor is given by: T= abvm/sec T=01X12? x1 xv m/sec And now, T= Q OLX 12? x1 xv =07 4.63 m/sec 36 —_Asimplex table shown below is generated during the maximization of a linear programming problem using simplex method Varate | 2 [| 2. 3] xa] mas Z a [aj ota of 6 i x2 o [4a| 1 3A 0 x4 o [alo 2a [a After one iteration, the value of the objective function becomes (a)4a/7 (8) 31/3 (22/7 (0)2/3 ‘Answer (A) Solution . Iteration procedure ‘Step 01, Select a most negative coefficient in row (0). Put a box around the column below this, coefficient and call this the pivot column ‘Step 02. Determine the minimum ratio test- 2) Pick out each coefficient in pivot column that Is strictly positive (> 0) b) Divide each of these coefficient in to the right side for same row. ©) Identify the same row that the smallest of these ratio, —— vate ans, sate ele 5 ice ¢— | —S EE ‘d}_ Puta box around this raw and callit plvot and also call the nc mber that isin both boxes the pivot number. z ° 1 oe ° 6 cy 1 o | wa Bo 2 x 2 o | wl o ae 4 2 ‘Step 03, Divide the pivat row by pivot number and obtained row called new pivot row. ‘Step 04, Performing operation ~ Row (0)-» Row (0) + Row (2) Performing operation — Row (1)-» Row (1) ~3 Row (2) aw eanee coattient of wenaoie | ea ae 2 nm | % z | 1 [oo 1 ° 6 ° 1 o {fp wft a wl oe 2 x a 2 | o {fmf weft 2 z o ft Cn) wan 49/7 2 % ajo lo 1 an ain | aaa A 2 fo ]fa ° ma “7 After one iteration , the value of the objective function becomes = 48/7 937 The value of ff log(cosx) dx is (A) ~1¢/2 logz (8) —1/4 log? (C) m/2 togz (0) 1/4 log? answer (A) Solution. Let, = ff" loge cos x de. 1 = Jf? tog cos (2) (2"* loge sinx dx. (2) [+ properties f° fax = fy f(a —x)dx) ‘Adding (1) & (2) 123 21 = I" loge cosxdx + f°" log, sinx dx 21 = J)""Ulog, cosx + loge sinx + loge 2 — log, Z]ex ape a= f loge(2sinx cos x) — loge 2]dx 21 = fy loge sin2xde ~ f°" loge 2 de 21 = Plog, sind xdx — Flog, 2 In first integral, putting 2x = 6 then d= 1,8 =Oand whenx = 2,9 = When x= £21 {loge sin 8d ~ Flog, 2 21 = 25 fy" loge sin 8d "tog, 2 2 = 1—Hoge2 —Floge 2 2.38 Given the following, Machine ‘Component P. Dint header 1. Cow! 0, Coal plough 2. Cutting chain R. Road header S. Shearer the correct match Is (A) P-4,0.2,R-3,5-2 (0) P2,03,R-4,5-4 18 Answer (0) 39 Giventhe following, Rescue apparatus Characteristic . Draeger BG-4 4. Open circuit chemical oxygen self-rescuer 0. MSAIW-65 2. Filter type self-rescuer R. Draeger Pulmotor 3. Self-contained breathing apparatus '. Oxyboks 4, Resuscitation apparatus the correct match is (A) P.3, 022, Ra, 5-4 (8) P-4, Q-1, R-2, $3, (C)P:3, 2, R-4, St (0) P-4,0-4,8-3, 5-2 ‘Answer (C) 2.40 Giventhe following, Equation/formula/law Application P. Bernoulli equation 1. Pressure loss in laminar flow of fluid Q.Poiseuille equation _—_2. Drag loss due to regular obstructions in fluid flow R. Bromilow’s formula _3. Energy conservation in ideal fluid flow $. Stokes law 4, Terminal settling velocity of fine particles in fluid the correct match is 23,R2, 5-4 (A) P.3, 0.1, R-2, 5-4 (BYP. (0)P-2,0-3,R-4, 5-1 (D)P.3,Q-1, Rea, $2 Answer (A) ws Q.41 Four psychrometric processes P,Q, R and S are shown in the psychrometric chart below. ‘These processes respectively represent (A) dehumicification, humidification, sensible heating, sensible cooling (8) sensible heating, humidification, dehumidification, sensible cooling (C) dehumidification, sensible heating, sensible cooling, humidification (0) humidification, sensible heating, dehumidification, sensible cooling Answer (0) Q.42 Given the following differential equation fy te e472 4 12y=0 the general solution is (a) y = de** + Be (8) y = Ae“ + Be“ (ly = Ae + Be“ (oy = Act + Be™* Answer (B) Solution. We have 126 dty dy fy 7 ®sray=0 dxé "ox 12; Inthe symbolic form: pty 47Dy+12y=0 0 (D? +7D +12)y {ts auxiliary equation will be - D?+7D+12=0 Hence, general solution will be- y= de® + Be 2.43 Given the following, Mining method Technique P. Thick seam extraction 1, Double unit face Bord and pillar extraction 2.Jet cutting R. Longwall face development 3. Inclined slicing S. Hydraulic mining 4, Half moon method ‘the correct match is (A) P.3, 0-4, Re2, Set (8) P-1, Q-4, R-2, 5-3 (€)P-3, 2, RA, $-4 (0) P-3, 0-4, Rea, $-2 Answer (D) Q.44 Given the following, Excavating/loading machine Transportation scheme uy P. Bucket Wheel Excavator 1. Mine tub @, Continuous Miner 2. Armoured flexible chain conveyor R. Shearer 3. Shiftable Conveyor S. Load Haul Dumper 4, Shuttle car the correct match is A (8) P38, 0-4, R-2, St (a) P-3, 0.2, R. R3,S-2 (9)P-3,Q2,R-4,5-4 (0) P-4, a4 ‘Answer (8) 45 Asub-critical subsidence profile is shown in the figure below. The points A, B, C, and D represent respectively the points of = anon — (A) zero vertical displacement, maximum tension, inflexion, maximum compression (8) inftexion, maximum tension, maximum compression, zero vertical displacement {€) maximum tension, inflexion, maximum compression, zero vertical displacement {0} maximum compression, maximum tension, inflexion, zero vertical displacement 128 Q.46 Answer (C) ‘The uniaxial compressive strength of a limestone sample is 80 MPa. The sample Is confined at a pressure of 20 MPa in a triaxial compressive strength test. Based on Hoek-Brown failure ert the maximum principal stress at failure in MPa is. (consider rock constants as m= 7.88, $= 1.0 and a 5) (a) 117.9 (5) 132.3 (0137.9 (0)157.9 ‘Answer (D) Solution. a7 Hoek: Brown failure criteria by: 4, = 05 + 04 (m2 + s)’ Where, 1,03 are the major and minor effective principal stresses at failure «is uniaxial compressive strength of the intact rock material im, sand a are material constant Therefore, 20+00(780% 2241)" = 157.9 MPa A of length Lis cut into two pieces to construct a circle and an equilateral triangle such that the combined area is minimum. The length of the wire used to construct the circle is 129 ee eee “Answer (A). Let total length of wire used to construct equilateral triangle and circle = L Then, triangle length ‘And circle length = L = Ly Now, calculation of equilateral triangle area— 130 Now, calculation of circle area Circumference = 2nr 2nr orr= Now, combined area ~ mat wo ar For max, or min. ° =0 she ane Now, length ofthe circle = L— Ly ait ent “ 131 Solving equation (2), length (LJof the wire used to construct cele Is acrscd Common Data for Questions 48 and 49: Pressure characteristic of a mine fan is given by =0.069? + 400, where P is the pressure in Pa and Q? is the quantity in m’/s. The resistance of the mine is, 0.19 Ns?/m®. 48 Themine quantity in m? (a) 160.0 (o)535 (9459 (0)40.0 Answer (0) Solution, We have, aR 0.069? + 400 = 0.19 x Q? Q= [82 = 40 m/sec 2.49 Anidentica fans installed in the mine to operate in series with the existing fan. Soh ‘The new mine quantity in m’/s's (a)75.6 (8) 56.7 (9508 (0)30.2 Answer (C) ‘When identical fan is installed in series with the existing fan, then new pressure becomes az P’ P = 0.06 X2xQ?+2 x 400 PY = 0.129? + 800 Now, P'= RQ? —0.129 + 800 = 0.199? 2Q 50.8 m¥/sec ‘Common Data for Questions 50 and 51 “The following observations are taken during a closed traverse. Side Length(m) WCB AB 100 90° Be 173 180" ca 200 330" Q.50 The dlosing error of the traverse in mm is ta)205 (8)20.5 ()2.05 (0)0.205, Answer (A) Solution. um of the latitudes of the fines = 3k.cos = 100 cos 90° +173 cos 180° + 200 cos 330° ZYL-cos@ = 0.2050 m ‘Sum of the longitudes of the lines :- ELsin 8 = 100 sin 90° +173 sin 180° + 200 sin 330° Yksing =0 Now, Closing error of traverse = JEL cos 0)? + EL = VO2050 +O? = 0.2050m G51 Thereduced bearing ofthe closing error in degreesis taya7.21 (6) 64.03 (104 (oo Answer (0) Solution. Reduced bearing of closing error = tan Reduced bearing of csng err = tan? (2 =0° ‘Statement for Linked Answer Questions 52 and 53: Economic analysis of an iron ore deposit reveals that the net value of the ore is related to the grace mined as shown below: Grade (Fe) ‘Net value of ore (Rs/tonne) eas 3200 | 60.2 1800 Q.52 Assuming linear relationship between the net value and grade, the break-even cut-off grade in% Feis 134 (ays2.2 (8)54.7 (58.0 (oy 622 Answer (8) Solution. Assuming the net value (Rs/tonne) and grade (9% Fe) relationship is linear then fan equation of form y = a + bx relating to net value (y) to grade (x) So, 3200 = a+ 6456 1800 = a + 60.2b Solving both equations, a= ~17800 b = 32858 Therefore, result is =17800 4325.58. But , at break even cutoff grade net value (Y)is zero, ie v=0 17800 + 325.582 17000 = 17000 = 54.7% 325.58 Eh 253 Assuming thatthe grade follows normal distribution with mean 62.7%, and standard deviation 10.0% (A portion of the standard normal distribution table is given below), 125 SO, ete 00 en 0,02 maT 0.0 ee. 06 0.72575, 0.72907 OFT 0.73565 0.73801 or 0.75803 0.26115 0.76424 0.26730 0.79035 os 078814 0.79103 0.79389 079673 0.79954 09 osiso4 Os1859 082121 0.82381 0.82639 Lo oxsiaa o8i37s osa613 osasay 0.85083, the percentage of waste in the deposit based on the break-even cut-off grade is, (788 (yn (288 (o) 21.2 Answer (D) Solution. From normal distribution, we have "X= 54.7% b= 62.7%, 0 = 10% #=-08 From normal distribution table, at 2 = ~0.8 ‘The percentage of ore in deposit based on cut off grade i 73.814% Therefore, The percentage of waste in deposit based on the cutoff grade is = 100-78.814 =212% Statement for Linked Answer Questions 54 and 5 ‘A4.6 mwide vein dipping at 80° is mined by horizontal cut-and-fill stoping method. The fillisto be placed in the stope along the length of 46 m and to-a height of 3.0m. Ifthe specific weight of the fill material is 15.86 kN/m? and the 136 porosity is 35%, under fully saturated conditions 54 The volume of water in the fill in m? is (a) 222.18 (8) 332.40 (c) 336.44 (0) 634.80 ‘Answer (A) Solution. porosity = maine own water otal volume of rock sample (35 __ volume of water 100” 46x46x3 35 «46 x46X3 e-volume of water= "S92 5055 100 + volume of water= 222.18 m? ass ‘The mass of solids in saturated fillin tonnes is (a) 820.00 (8) 804.10 (c)799.30 (0) 788.80, ‘Answer (8) Solution. __ weight of solid Specific weight ume of solid weight of solid 1880 TG oxaexa weight of solid= 10076.928 kN = 10076928 N mass of solid = 1026292.355 ke And mass of water= 222.18 x 1000 = 222180 kg Hence mass of solids n saturated fill 1026292.355 ~ 222180 804112 kg 137 Qs6 Solution. as7 ass = 804.1 tonnes Me3SX<5 and 8G>K ( E=6>K (0) E>K>G ‘Answer (0) Solution. 146 mE, 3-2) Where, Bulk modulus = Modulus of Elasticity 4 34-2) k Or E=TSK cannon Ai) Now o- 2(m+1) where Modulus of rigidity 45 2 ABB 2g 8° 3asi) 2x55 £-256 i From {i) & (ii) «i Hence, from equation (i) & i) it is seen that, 147 EDK 6 The 2 order diferental equation having a soltiony=(A/xtB, where Aand B are constants 36 ey (0 Satie ) ey ey yy st + + Zao Answer (8) Solution, Given differential equation Asp Now ditterentiate wert, x, we get =) Senne Ai) 2A_ 2a 148 a7 Solution, as 2ay dy x dx dx? ‘Aclindrica rock specimen is unlaxially loaded under compression and falls at SO MPa. The {fracture plane is inclined at an angle of 45° withthe axial direction. The normal and shear stresses respectively on the failure plane in MPa are 18)50,50 (8)0,50 (€) 50,0 (0) 25,25, Answer (0) Shear stress given by- ra%=2=25MPa Now, += a, tanO+C When C=0; Soqtand Given that, 6 = 45° Ina surface mine, sound pressure level at location generated by operation of a dozer and adil respectively are 80 d8A and 60 dBA, when operated independently The sound pressure generated by the dozer compared tothe drills higher bya factor of (ay10 (@)20 (e100 (0) 200 Answer (A) 149 9. Asperthe indian Electricity Rules 1956, the maximum permissible length ofa flexible cable ‘used with an electric rope shovel in mis. (4) 200 (8) 200 {300 {0)500 Answer (C) 1.10 The equipment that s NOT used in hard rock metal mining drivage is (A)road header (8) drilljumbo (C)jackhammer (0) dintheader Answer (0) Q.11 The roof bolt that follows the principle of point anchorage is {A) expansion shell bolt (8) full column grouted bolt {C) pit set bot {0) swell tote Answer (A) 2.12 Equipment used in mining of placer deposits is (W)auger {8) wagon dil (0) rope saw (0) rtf box Answer (0) 0.13. Adump truck powered by 350 KW engine is running ata speed of 35 km/h. Considering the ‘transmission efficiency of the truck as 85%, the rim pull of the truck in kN is wa (531 aa (0) 51 Answer (8) Solution, 150 work done lm pull x distance or powers a Or power = rim pull x speed 5x0? 350 103 = Rim pull x 60x60 350510 60:60 35x10 fim pull = im pull = 36000N But given transmission efficiency is 85% im pull = 36000 x 0.85 im pull 30600 N rir pull = 30.6 KN or 31KN Q.14 Nystagmus isa miners disease associated with (a) lever (8) ung (eve (0) stomach Answer (C) @.15 Apart from mining of coal, the longwall mining method has been practiced for mining the deposits of *(ayeorper (8) lead and zine (C) manganese (0) pyrite and phosphate Answer (0) asi The three segments, whose synchronous functioning is essential for GPS operations, are (A) space, control and user (8) signal, control and user (C) space, control and geo-registration (0) signal, control and geo-registration Answer (A) When a double ended ranging drum shearer cuts coal In alongwall face, (A) both the drums rotate in the same direction keeping the front drum up and the rear drum down 8) both the drums ratate in the opposite direction keeping the front drum up and the rear drum down C) both the drums ratatein the opposite direction keeping the front drum down and the rear drum up 0) both the drums rotate in the same direction keeping the front drum down and the rear drum up answer (8) ‘The match the following, Mine gas Principal constituent P. Stink damp 1.00 Q. white damp 2s R. Black damp 3.cHy S.Fire damp 4.00; (8)P-1,02, 23,54 (6)P3,0-4,R4,52 (0)P2,04,R.4,53 (0)P2,04,R3, answer (C) 152 Q.19 Continuous miner and shuttle car combination is NOT applicable in mining with (A) rib pillar extraction technique (8) Wangawill system (0) longwall method {0)room and Answer (0) 20. Contours ina topographic map (A) are not closed upon themselves although the earth isa continuous surface (8) are not perpendicular to the direction of maximum slope {C) provide an indication of presence of valley or ridge in the area (0) are the lines joining the points of same declination at different elevations Answer (C) 21 Dragger Gas Mask DOES NOT filter (A)water vapour (8) nitrous fumes {Clearbon monoxide (0) carbon dioxide Answer (8) 0.22 Asystem consists of four elements A,B, Cand D which are connected functionally ina parallel configuration. The individual reliability ofthe elements is 0.80, 0.82, 0.85 and 0.90 respectively. The reliability ofthe system is (a)o.98, (8) 0.602 (0) 0750 {0) 0.999 ‘Answer (0) Solution. Given that, 153 923 a2 Relibiity of element. Reliability of element Reliability of element Reliability ofelement B ‘The reliaility of system will be R= 1~(2-A).2~8).(1-0).(1=0) R=4~(1~0.80}. (1-082). (1-085) (10.90) R=0,999 Ans, ‘The blasting technique used for controlled throw of overburden is known as (Ajcast blasting (8) coyote blasting (C)plaster shooting —_(D) pop shooting. ‘Answer (A) ‘The stoping method, where a large part of blasted ore is allowed to accumulate inthe stope (o serve the purpose of providing working platform for stoping as well as to support {the watl-rock, is known as 14 {A) shrinkage stoping {8) cart and fill stoping (C)squaresset stoping (0) sublevel stoping Answer (A) 0.25 The Injury rates of mine workers in an underground coal mine based on age group are sven helow: nN csstnmewetens | Again Age spose pomsotonsa teaming as 0 ‘Th injury rate per 1000 persons employed inthe mine forthe total population fs (ajoza (ey2aa (c) 8.80 (0) 24.40 Answer (8) Solution, Injury rate per 3000 employed in the mine forthe total population Is 1.821000 2.5x500+4.5%300 *1000+500+300 26 _Ashearer is deployed in a mine where the specific energy consumption for cutting coal is £800 ki/m’, The specie gravity of coals 1.2, Ifthe machine produces 700 te/h, the electrical power consumption in kW of the shearer at 65% motor efficiency 155 (9.4 (@)199.4 (@ 294 Answer (8) Solution, az {1m* coal consume = 800 Kd 111.2 tonne of coal consume = 800K) 00 1 tonne of coal consume = Se 700-900 700 tonne of coal consume = 12 tonne 700 » £00 70022" of coal consume = “22% 500) ® ah 700x800 xy 2% 3600 © or 700 2H of coal consum 28H of coat consume = 722~200 4s * 12% 3600 + or 700) or 700222 of coal consume = 128.63 kW 6) 129.6 048 Input power = 7253 - 199.4kw (0) 239.4 ‘The figure shows a weightless beam PQ of length 8 m resting on a hinge support at P and ‘on atoller support at R.A vertical force of 40 Nis acting ata distance of 4 m from P. A uniformly distributed load of 10 N/m s acting on a length of 2 m of the beam from Q. 156 “The magnitude of reaction force at Rin Nis. (a)20 (®)30 (40 (0) 50 Answer (0) Solution, | 1onien | | x | i \ Ll an a al am ad =M 40x84 Rx6-10%2%7=0 140+160 6 R=50N 28 The gear ratios ofthe first gear, transfer case and differential ofa four wheel drive vehicle ‘are 3.81:1, 2.72:1 and 4.11:1 respectively If the engine is rotating at 1000 rpm and the a7 ‘wheel diameter is 1.2 m, the speed of the vehicle in first gear in km/h is Difterential (est BOX Transfer Engine Case ja (aysan (2) 3.68 (eae (oy 468 Answer (A) Solution, CGreumstance of wheel= 20 =344123.77m Engine spr = 1000 rpm we have, Vehicle speed emp awl reference Vehicle speed SORLRAAL A 18.47 fminute aes 100k = 5a keh Hence speed of vehicle in fst gear I = 521 Km/h 0.29 Anion are mine recorded an average of aecilents per month. The numberof 158 accidents is distributed according to Poisson is tion, The probability that there: be exactly 2 accidents per month is (ayo.2z {8)0.30 {(C) 0.43 {0) 0.67 answer (A) Solution equied probabity = Gen that, ‘ity «2 0F. Reade protabity = o22 2.20 Moteh the folowing: Equipment Component P.seraper 4. Dribble belt a.oragtine 2. Dipper stick AR. bucket wheel excavator 3. Fairlead 5. Rope shove 4. Bowl (A) P.2, 04,83, 54 (0)P-4,03, Re, $2 Answer (C) 12) P-4,0.2,R4,$-3 (0) 2, 04, R-4, 53 31 The torque in Nem of a winder motor is described by the relationship T= 1450- 3.20, ‘where, wis the angular speed of the motor in rad/s. ithe shaft is rotating ata speed of 1450 rpm, the power of the motor in kW is 159 (aa (@)146.4 (o) 188.8 (0) 2124 ‘Answer (B) Solution Given, T=1450-32.0 2h, aa/see where, N=rpm 2x aad 1450 51.7 radfsecond Now, T=1450-3.2 151.77 T= 964.336 Nm Since P= 07 P 2151.77 x 968.336 P=146.35 KW ‘An investment of Rs, 10,000, compounded annually Is estimated to return Rs. 20,000 an after 6 years from the date of investment. The expected rate of return on this investment In percentage ts ayazs ()1050 (0 1225 (016.6 Answer (C) Solution =P(+iy® Given, $= 20,000, P=10,000 160 ax Solution, aaa nes 20,000 10,000 P(t vi) ir The viscosity of air is 4.85 x 10° kg/m s, Neglecting ai density and assuming that the ‘settling of droplet in air follows Stokes’ Law, the settling velocity in m/s is tayo.98x10% —(u)2.95 «10° {9 s0ax1n* (0) s3.08x105 ‘Answer (8) According to Stoke's settling velocity is given by 229-0 at) Fs py = 284% (40°) 1000 Wx 85% 10°F v=2.95x 10 m/s ‘Amining company has three mines (MA, M2 and M3) that supply coal to three power plants (P2, P2 and P3). The three mines produce 800, 1000 and 1200 te of coal per day respectively. The power plant requirements from these three mines are 1200, 1000 and 161 {900 te per day respectively, The unit cost of transporting coal from the three mines to the three power plants in Rs. is given below Power plants PL 92 3 ML 8 10 2 m2 22 B 2 Mines Ma 16 10 u Based on the initial basic feasible solution, using Vogel's approximation method, the total transportation cost In Rs. is (a)31200 (@)31400 {o) 32800 (0) 40000 Answer (8) Solution. Given that, Power Plants Supply a A Ps Me 10 i | (90) Mines MD 8 Rn 1000 Moa 10 i 1200 fequvement m0 1000 500 Total= 3100 Now, for easy in calculation, taking scale of 100 = 1, for supply row and requirement column, then 362 12.3) 7(12) 10/7/0 3/0 70 10 ‘Therefore, 163 -3(900) | ow 12300) 2700) | 120 14(200) | 1200 x0 100 Total transportation cos % 900 + 12 « 300 + 12%700-+ 10 x 1000 + 11.» 200 1400 Hence total transportation cost is Rs. 31400 0.35 The angle between the tangents to the curve 2= 774217 at the point t= 41 is ye (oy n/3 toma (orn ‘Answer (A) Solution. Given, R= eT 4207 andt=e1 (ee ee N on vae(I2 23) e ia}) y, U2 Nowat,t=1 (a.R) <2r+ Now att 168 (8.8)y =-2re21 since, _(R) (9-8), cop 21 W8), IVR] IVR, (2t+-25).(-2142)) 2742 2)? 422 cost = 36 The chip sampling data, spaced irregularly fora god vein deposi, ae shown in fgure. The sample points have equal influence on both the sides. Distance of sample from starting point(m) «210.6 Wiath (em) 70.80 60100 assayeite) 5 7 6 4 “Troe fw Ie) Sampling distance —> 2m 8m 10m 16m ‘The mean assay value in g/te is (wyos2 (@)5.50 (c) 5.9 (0) 4.50 165 Answer (C) Solution Paine] Widh(al] Asay | Onancel | devel [frase sande Iidt_ | inence | iene | rede re ,s, s | s | i Sarees jeer cet a2 | 78 fw [oe [is [ « [iw | re 6 | 4m t m | se | tea | ma areaassay product = "Yierea of inytuence Mean assay value ~ 5.9 g/tonnes Jal tests of sandstone samples reveal the cohesion and the angle of internal a3? Aseries oft ‘ietion 2521.65 MPa and 30° respectively. Based on the assumption that the sandstone samples follow the Mohr-Coulomb's failure criteria, the tensile strength in MPa is (ay1250 (1875 (0) 2165 (0) 25.00 ‘Answer (0) Solution. Tensile strenath (o,)= 2 S22 16 2.38 The adjusted values of departure and latitude fora traverse line AB obtained in a feld survey of a mine are 225.520 m and 388.835 m respectively, The length in mand azimuth of tine AB are (4) 429.50, 30° (2) 610.6, 30.37 (0) 614.36, 45.42" (0) 499.50, 45.1" Answer (A) Solution a0. Latitude = L c030 Departure = 1 sind Where, l= Length of tine = Azimuth of tne Given, Latitude of ine AB. 88.835 m Departure of ne AB = 225,520m Now, x sin6 7 Ai) Now dividing equation {) by equation i, tant oar? 167 Putting value of 8 in equation (i), we get ‘388,835 = 1x eos30.11° 449,50 m, 0.39 The figure shows the values of seven perpendicular offsets and the respective locations along the line XY as observed while carrying out a traverse survey. The area ofthe plot an 0.00 AD HTB. {all measurements are in meters) (a)26.20 (8) 43: (oyaasz (0) 62.50 Answer (C). Solution. The area of plot XABCOEFGYis= 05 x(7.24119) +F08 (144 109)45 42 2 assed 403% (64 6a) +E 10508464) +5 06x (18 4124 7341052 +102 +1.813 + 9.3975 47.26 =44.92 m? 168 240, Ina longwall pane! the main gate road is 1000 m fon, 4.5 m wide and 2 m high. The gate road Isto be used for airflow at the rate of 17 m/s. Considering a coefficient of resistance of airays of 0.01, the pressure Pa required to maintain the arflow in the gate road Is (a)sa.33 (0) 463.84 (© 375.98 (0) 7885.32 Solution Pressure is given by Given that, (01, $= Length x Perimeter 3 1 2 Q41 — Thecofactormatixot P=|2 3 4] is 12 3) a5 2) 2 w [2 ot 2) off 3 3| oF ot 169 s 2 its © [e 7 2 (0) Fs 2 c Solution. Cofactor of element 3is = ( ya \2 Cofactor of element 11s= (-1)'*2 | Cofactor of element 2is = (-1)!*9 | Cofactor of element 2 = (~1)?*t 3 cofactor of element ais= (1) |? Cofactor of element 118= (1) E ae 72 2 al a 3 2) a 23 4, a 3 1 2| 1 _ ramvtosororwisrs [I 7) 5] aa Match the following: Mining system Face supports . Mechanized fongwal in flat seam 41. Cable bolting 2. Blasting gallery method 2. Shield support R, Mechanized longwallia steep seam 3. Alpine breaker ine support '. Wangawill method for3 m thick coal seam 4, Troika shield support (a)P2, 0.4, 8-4,5-3 (6)P-4,04,8-3,52 ()P-4,02,R3,5-1 {0)P-2,03,R-4,5-4 Answer (a) 43 —_Anopencast mine bench has a potential failure plane AC as indicated in figure. Bolts are installed to stabilize the fallure plane providing a resultant bolting force of 300 KN. The ‘area of sliding block ABCs 37.45 m’. The unit weight, cohesion and angle of internal fiction of rock are 25 kNi/m*, 20 kPa and 40° respectively. ‘The factor of safety of slope when bolts are installed perpendicular to the failure plane is an (ao79 (8)2.08 (ave (0) 3.46 Answer (8) Solution. CA (WW cospeT)tand WsinB where, cohesion 10 KPa; = 80" ; P= 70° sare 1936.25xsin70° Fos=1.08 44 Figure shows a two pulley system for hoisting a laad of 10 KN. The coefficient of friction between each pulley and the rope is 0.2. The vertical and horizontal distances between m the centers ofthe pulleys are 25 m and 16 m respectively We tokn Th ‘The tensions T; and 7 respectively in kN are (4)6.09, 5.38, (8) 12.37, 11.06 (c) 18:74, 16.73 (0) 25.11, 22.41 ‘Answer (C) Solution, 20KkN Here Given, w= 02 rst 5 Fy ow, tan = 2% i 0=57°22's1" - Angle between T, & Tp willbe 0 =90°—57%22'51" = 32437110" Or 6, *0.57 radian Th pyyg02057 Tz Ty 2112077, .. sl) 174 = 323710" Angle of ap between Ty & Ty is =90"+ 0 |.0,090?-an%ars0'a87°205r| 47°22'51" = 2.57 radian _ Angle of lap between Ty & Ty is = 90"+ 5792'S 2 Ty _ 971902287 70 ‘ty “167106 From equation (i), Ty #142070 16.71 Ty #18.73KN us ‘Common Data for Questions 45 and 46: ‘A2.5 m thick coal seam lying at an average depth of 100 m has been developed by bord ‘and pillar method. The width of the square pillars is 30 m (centre to centre) and the tzallery width is 4 m. The average deasity ofthe overlying strata is 26 kN/m3 and the pillar strength fs 4500 kN/m?. 45 Extraction ratio during the development ofthe pillar Is (ayoans (0.108 (oz (0)0.289 Answer (0) Solution, | (a+b) Square Pitor i i a=30-b=30- 176 2.46 The safety factor ofthe pillar is yt (13 (as (0) 47 Answer (8) Solution. Factor of safety =—St7enath of pillar Toad acting on pillar Pillar strength is= aSO0KN/m? = 4500 103 N / m2 Load acting on pillar Is given by P=Pox 1 or pswdx 1 Pe w=26kN/ m?= 26 x 103 wm? @=200m Here, P = 26% 103x100 —! 0289 P = 3462050599 N/m? 4500108 % Fos = 450010" 162050599 Fos ‘Common Data for Questions 47 and 48: The following data are provided for a surface mine to be excavated by a shovel: 7 Production target + 10000 te/shift Available hours per shift 26hrs ‘Shovel loading cyeles per hour 106 Bank density of the material mined :2400 kg/m? Swing factor at 120 Cswing 10.91 Bucket fil factor 10.68 Utzation of availabe time 89% ‘No. of working days ina year No. of shifts per day Q.47 The annual production target in Mite is, (as.76 (@)7.00 (aas (0) 9.00 Answer (0) Solution. ‘Annal production target willbe = 10000 x 3 x 300 = 9000000 tonne =aMte Q.48 The size of bucket of the shovel nm" is (ass (69.33 (c) 12.22 (0) 1355 Answer (0) Solution. Productivity of Shovel 3600xCaxarx x7. : 7 mih where, 178 Cd Size of bucket, m? F-Fillfactor ‘= Swing factor (> Cycle time, See 11-Percentage of utilization Here, A= 10000 ysnit= 100° gynour = 100001000 gg ipour 6 _ 100001000 3, 2499 /hour tg thou 06 oy 106 cycle =1 hour 1 3600 teyele= 1 hour 562 seconds v 106 106, vs Putting all values, 1000x1000 _ 3600xCax0.64x0.91x1x0.83 62400 3600 { 106 can 100001000 (6 2400%0.64%0,91x0.83x10° ca=1355 m? 179 Statement for Linked Answer Questions 49 and 50: ‘A mining project is composed of ive activities whose three time estimates in months are aiven below: xmsted duration (onthe) pumice voit lelytime |] Permissible te ry 2 5 “ Q.49 The expected duration of the mining project in months is (ays. (9/16 to38 (0)29 ‘Answer (C) Solution, 180 Aaivty to tn fot ate +t " » fa a 1 32 5 8 5 1 ee a 1 a2 8 6 4 ioe ce as 7 4 ence various path through the network ad ther length is glen by- 1-24-52 24247211 1-3-4-5254 647018 Since path 1 -3—4~Shas longest duration, itis the eitical path of network Expected project length is= 18 months. 50 The standard deviation of the project length In months is we (3 os (9 ‘Answer (8) Solution. Variance = Vy_3+V3~4+Va-s Veltasaes But v=o" aa ‘Statement for Linked Answer Questions 51 and 52: In amine between upcast shaft and downeast shaft, two airways are connected in parallel and their resistances are 100 Ns*m" and 120 Ns%m® respectively. The resistance of upcast shaft, downeast shaft and the fan drifts are 10, 20 and 5 Ns*m respectively. The fan deift air pressure is 15 MN/m?, 51 The rate of airflow through the mine in m'/sis. (sas (8)38.26 (0240.5 (0333.33 Solution, wt R=27.33 NS? “Total resistance of mine = 27.33 +20+10+5 =62.33 NS?m* ae 15x 108 62.38 « Q? fisx10® oy azas Q=490:57 m°/see 52 Therate of airflow through the split alway having resistance of 100 Nsm*inm’/sis woaz (wyo79 (ozs (0) 790 soluton Rate of air flow through the split air way having resistance of 100 NS 2m 8 willbe ay-ax [E VR p38 aso PE co Vi00 = 256.46 m/s 253 choose the most appropriate alternative from the options ven below to complete the following sentence: |_to have hought a diamond ring. (a)have aking. (0) should have liked {C) would like (0) may tke ‘Answer (C) 0.54 Choose the most appropriate alternative from the options given below to complete the 183 ass. ass as7 following sentence: Food prices __ again this month, (A) have ralsed {B) have been raising {0 have been rising (0) have arose ‘Answer (C) {Choose the most appropriate alternative from the options given below to complete the following senten The adm rators went on to implement yet another unreasonable measure, arguing that ‘the measures were already __and one more would hardly make a difference. (A) reflective (8) utopian (©) huxuriant (0) unpoputar ‘Answer (0) ‘Choose the most appropriate alternative from the options given below to complete the following sentence: ‘To those of us who had always thought him timid, his__came as a surprise. {Ad intrepicity (6) inevitabiity () inability (0) inertness ‘Answer (A) ‘The arithmetic mean of five different natural numbers is 12, The largest possible value among the numbers is, wr (@)40 (950 (0)60 ‘Answer (C) 184 Solution. Let five natural numbers are x1, x2, X43, 4, X5e then Ky PX) AR ERY ERS R= 5 x1 + x2 xg + x44 x5 =60 Take, xy #50, x2*4, x3 #3, 2492, 25° ‘Then, 504443424160 60=60 Hence, largest possible value among the numbers i- x1=50 ‘Two policemen, Aand 8, fre once each atthe same time at an escaping convict. The ase Probability that A hits the convicts three times the probablity that 8 hits the conve: the probability ofthe convict not getting injured i 0.5, the probability that B hits the convietis (ayoaa (e)022 (0 033 (o}0.40 Answer (A) soltuion. P(a)=3016) and (AUB Since P(AUB)=P(Al+P(8)- P(ANB) Here, (ANB) P(AUB) = P(A) + (8) 185 05 =3P(8) +P(8) P(e)=024 59 The total runs scored by four cricketers P,Q, R, and: years 2009 and 2010 ate given in the following table: Player [2009 2010 Ema P 802 1008 a 765; 912 R [429 619) 5 50 701, ‘The player withthe lowest percentage increase in total ninsis we (0 cor (os ‘Answer (8) Solution, For player ® percentage increases in total runs 1005-802 399 = 20.3% 1008 For player Q percentage increases in total runs 12-765 912 100 = 16.12% For player R percentage Increases in total runs 186 619-429 619 For player S percentage Increases in total runs 701-501 = TEST 10 701 8.53% Hence the player with the lowest percentage is total runs is Q, Ans, 60 Ifa prime number on dvsion by 4 gives a remainder of 3, then that number can be expressed as (a) sum of squares of two natural numbers (8) sum of cubes of two natural numbers (€)sum of square roots of two natural numbers (0) sum of cube roots of two natural numbers Answer (A) Solution. Let prime number i 5 and when this prime number is vided by 4then gives remainder of Let two natural numbers are2, 1 and hence prime number 5 can be expressed assum of squares ofthese two natural numbers Le eee aries 5-5 187 61 Two points (4p) and (0, a ie ona straight ine having a slope of 3/4, The value of (a) aya {8)0 (3 (oy a Answer ( Soliton. Vem Cn 1) Given two pons (4, P) and (0,4) and slope of 38 Now, a point (4) and slope 3/4, equation becomes P= Snare 4 P=34C, von) Now a point (.9)and slope? equation (becomes, asaxore VEC css iii) from equation) a (i, is found that 262 Inthe cay inetonth entry teas of oa eveuon wer nsprdes by Baoey ‘han by the conviction of social slentists that there was a growing improvement In social Institutions. Progress was taken for granted and soca sclentists attempted to discovers Jaws and phases, Wich one of the following inferences may be drawn with the greatest accuracy from the 188 above passage? Social scientists {A) did not question that progress was a fact. (8) did not approve of Biology. {C} framed the laws of progress. (0) emphasized Biology over Social Sciences. ‘Answer (A) 189 1 A scatter plot prepared using.a set of values of lead and zinc from a lead-zine deposits. shown in figure below. The value of correlation coefficient is ine (4) oem tayo (07 (05 (yo Answer (A) Solution. 8y looking at the scatter points, itcan be seen that all ofthe points are very close to being along a straight line So there is a perfect positive correlation between lead and zinc from lead tine deposit Therefore, Correlation coefficient (.)= +4 @2 — Thevabeot lim, 9} (VTFE—VI=H i wo ea (2 (03 Answer (8) Solution. re) lingo (VTP VT= 3} 190 FO) = Vip} ES x VIF + VRS} 10) = if EEE 70) = lime £0) = lima 3} £0) 3 Theinnteseres 14244 ou (A)convergent_{B) divergent _—(Choscllatory——_—(0) semi-convergent Answer (A) Solution. Gh noses, 14212124 or Si Sor =E2, “7 <1, the series is convergent. Q4 —_Thelargest area ofa rectangular shat fora given constant perimeter i obtained when length is {8)25 times of breadth {6) 15 times of breadth (0)2 times of breadth {0) equal to breadth ‘Answer (0) Solution Perimeter for a rectangular shafts given by:- a9 Perimeter = 2{length + breadth). vol) Let take perimeter isx From equation (1) length + breadth = 05 x an. 2) Case -1. When length is 2.5 times of breadth from equa. (2) 2.5 breadth + bread osx « breadth =0.14.x length = 0.36. ‘Area =0.0508 x* ‘Case -2. When length is 1.5 times of breadth from equa. (2) 1 breadth + breadth = 0.5 x breadth =0.2 length =0.3x LArea =0.082% case 3, When length is 2 times of breadth from equa. (2) 2 breadth + breadth «breadth =0.17 x slength = 0.34 x Area = 0.0578 x? ‘case-4, When length is equal to the breadth from equa. (2) breadth + breadth = 05 x ae breadth = 0.25. s length =0.25 x «Area £0,063 x? Hence, In case of 4, Avea of rectangular will be largest. 5 drive shaft ofan engine develops torque of $00 N-m.ftrotates ata constant speed of 50 rpm, The power transmitted by the shaft in kWis (w)aas (3)2.05 (o) 262 (0) 432 Answer () Solution Relatian of Power (?),Torque (7) and Rotational Speed (w) are given by- Prot 22st 5909 2 P = 2616.7 watt P= 262 KW 6 —_Armine winder cage traveling 450 m from pit bottom to pit top i following a three period duty cycle as shown inthe Figure below. The maximum velocity attained by the cage in m/s is a (7s (90 (9 10 (0) 120 193 Answer (8) Solution. Here, at point B, the cage will attain maximum velocity. Hence, maximum velocity attained by the cage is 0.7 Stress concentration at a point on the wall ofa vertical shaft results in a compressive stress of 59.66 MPa, The wall rock mass has an unconfined compressive strength of 89.49, (MPa. the safety factor of the shaft wall at the point is (ajos7 (8) 0.86 (c) 1.23 (0/150 Answer (D) Solution, safety factor = Safety factor = 1.5, 198 8 —_Acore sample of $4 mm diameter having Young's modulus of 68.97 GPa fails in uniaxial compression at 0.1% axial strain. The axial load at failure in kN Is (a)3s800 (8) 68.97 {58.00 (0) 15.80 Answer (A) Solution - We have, stress= young modulusxstrain or A¥iet — young modulus x axial strain 1x10] Axial oat. 69,97 x 10° x 1 x 1079 fv Axlal strain = 0.1 ct 9.97 x 10? x 1 x 10°3 x 3.14 x (27 x 10°9)? N Axlal load ‘lal load = 157876 N ~ Avil load = 158 kN 0.9. The maximum number of coal faces in an underground bord and pillar development dstrict is 13. The number of headings inthe district is (as (os as (07 Answer (8) Solution 195 Barrier From above figure, relation between number of face (mn) and number of head by: m=axn-2 13-30 2 nes 2.10 The whole circle bearing ofthe line AB is 116° 20! 20) 20, the true quadrantal bearing of line AB is (A) 541° 59°40" (6) 543° 39'40"F (0) 545° 59'40"W (0) 547° So'40"w Answer (8) Solution. 196 ing (n) are given there exists an east declination of TIM. = True Meridian MAM. = Magnetic Meridian True bearing of ine AB = 116°20'20" + 20°0'0” = 136°20'20" ‘True quadrantal bearing of fine AB = 5 (180°0'0” ~ 136°20'20") E «True quadrantal bearing of ine AB = $43°39'40" E @.11 tls proposed to connect two straights ofa road by a simple crcular curve. ithe maximum, speed of the vehicle Is 60 km/h and the centrifugal rato for the road Is 2/4, the minimum radius ofthe curve in m is (wyasa.26 (0988 (0) 25.46 (0) 1550 Answer (a) Solution. We nave, Centritgat ratio= © = Ww" oR Where, P— centrifugal force W- weight of vehicle 197 4_ 667? a omxe R= 113.26m @.32 A centrifugal fan rotating at 500 rpm delivers 70 m'/s of air. I the speed is reduced to 200 rpm, The quantity of air delivered in m/s will be (yas ()55 (c) 28 (yaa Answer (C) Solution We know, Quantity of air (Q) delivered varies directly as fan speed (W) aan Oa G20 Q2=28m"Jsee 13, According to mine regulations, the value of the fleet angle a, in degree of a drum winder installation lies in the range of, (W)is -2 60/0 sis s(s0 Ce 0/30/10 Demand so/0 xzo/uo/s0 30/0 ence , transportation cost = 7 x 50-+3X10-+4x 60+ 6% 50+7 X30 130 @.26 high volume air sampleris operated for 8 hours in amine with the flow rate of ai varing fcom 15 m3 min to 1.3.13 min The empty weight ofthe filter paper's 2.30.8 and the final weight i 2.65 g- The mean concentration ofthe suspended particulate matter (SPM) ing the study period in pig/m* ‘ (ayson (@) 550 (0) 545 0) saa ‘Answer (0) Solution Total suspended solids/suspended particulate matters given by > agi? Tss/sM gl’ 2.65 x 10° 19) where, A- weight of iter plus retained solids in Hg =2.65 6 30 gm=230% 10° 49 B- weight of clean filter in ng volume of sample filtered in mn? 255 Am¥fminute Bhour = 8x 1.4 x 60m? Therefore, 108 = 521 yg 2-37, Aeveloped panel fora coal seam having incubation peri of 6 months has 22 square Pillars under extraction having size 25 m, and height 3.0 m. Density of tha coal is 1.4 fonne/im?. Extraction ratio during depillaring is expected tobe 75% To depilarthe panel ‘within the incubation period, assuming 25 workings days in a month, the production from the panel in tone/day is 1ayaz0 (8) 480 (0) S60 (0) 680 Answer (A) Solution. Production from one pilar= 25 x 25 x 3 x 1.4 =2625 tonnes So, production from 32 pillars = 32 x 2625 = 84000 tonnes * Production of panel within incubation period (6 x 25 = 150 day's) = 84000 tonnes Theredore , Production from panel wth 75% percentage of extraction in tonne/day is:- S075 = 420 tomes fay 0138 Adlosed traverse ABCDE of perimeter 425 m has a total error +0.25 min latitude and ‘0.44m in departure, The precision of traverse is (A)1ins56——(6)11n775.—(c) tin a3 (0)1in 2024 256 ‘Answer (C) Solution. ‘We know, Closing error = atteudey? + Congitude)? losing error = O25)" + (044 Sim Closing error = Sinee 425 m has a error of 0.51 m Am has a closing error of 2 or SEO as Hence , precision of traverse i 1 in 833. 39 Thevalue of tho given integral is fo sinx Me (Ginx + cos.x) sivas = snags a0 wt es (9 S38 oe sohtion - 2) ‘Adding (2) & (2), 27 2.40 The probability of hitting a target by A and 8 are 1/3 and 2/5 respectively. A shoots atthe target once, followed by 8 shooting atthe target once, The probably of iting the targets wns (8) shs (ans (©) 91s Answer (0) Solution. et, pia) =1/3 & pl) «2/5 The probability of iting te target is- = IA) pla) + pla) iB) + pla) pia) pet pe(t-$)+ (1-3) po? 41 _Thovalue of for which the point (5,5) (k.{20,7) lie on a straight tines w-s (+5 to -2 (os Answer (A) Solution Equation of nei given by : = 258 Where, m- slope ofthe line 6 the intercept of line Now , for point (5,5) seine a for pt oe for pnt 1.7 710m +c 9 sig) 80, from equation (2) Q.t2 steel wice ropes of 25 mm diameter weighing 37 N/m has & strands of 7 wires each, The diameter and tensile strength of each wire are 2.5 mm and 1800 MPa, respe ly. The factor of safety for raising a cage of weight 60 kN from a depth of 200m fs (45.60 (0450 (cy4.25 (oyaas, Answer (A) Solution. We have, Strength of rope Factor of safety = Sirenstnoe rope tor of safely = “iyeignt on rope Now, strength of rope = (6 7) strength of wie x cross sectional area of wire 9 14 ye OSX strength of rope = (67) x1800% 108% 3.14 x SSA200" 259 Strength of ron 370912.5 N ‘Weight on rope = 60 x 10° +37 x 200 Weight on rope 67400 N Factor of safety =55 43 In block caving operation the draw points are placed at 20 m center to center , with the pillar width 3.5 m as shown inthe figure below. The muckis assumed to have zero cohesion and 35° angle. The height of draw cone (h) in m is wis (e)146 (sa (o)165 Answer (C) Solution. 260 In Babe, tan275t = 828 nessa Common data for questions 44 and 45 : “The granular media in ore bin s assumed to be of regular spherical shape represented by the geometry as shown in the figure. The unit weight of solids is 25 kN/m?- 0.44 Thevoldratlo (ayosa (e084 (o7s (0)0869 Answer (A) Solution. vote of 30d Void ratio= Volume of sold = 8 xx 1 = fac xD? . = 004 —fxnxo? avt-tenxo? Void ratio TxaxD? Void ratio = 0.91 261 45 _‘Thedry density in kN/mn? is inno (oi2s0 au svar ease axticnxt += 104.67 D® Dry density volume aD? Mah sane ot singe ae shen nee ow | 60° 146 tetagettenethin mis Wass zea gama yon 262 (o)1087 Ansuer(0) Sohaton. wehave, Tangent tenth (of simple cuve = Rtan$ here, Ras of curve sb ttersecton angle 1 = 400 tani 1 =2309m .87 The length of the long chord in mis (ais (0) 450 (aya7s (e400 Answer (8) Weave, Length of tong chord (C) 6 =2x 400xsin ¢=400%m ‘Statement for linked Answer Questions 48 and 49: ‘Alongwall panel with a face height of 3 m and face length of 150 m i depth of web of the shearer cutting coal is 9.5 m. The is worked in3 shifts per day employing 40 men per shift. The: 1148 tonne/m?, Two full face cuts are executed per shift. unit weight ofthe coal “The daily production from the panel in tonnes is aaa, (3890 (o)2530 {ay 9as (8)3240 263 Answer (C) Solution . Dally production of shearer in tonnes numberof shift) x (two full face cuts) x (face length) x (face height)» (web depth) x (unit weight of coal) Daily production of shearer in tones X 2X 150 x3 x05 x 1.4 tonnes ‘= 1880 tonnes, 2.49, The panel OMS in tonnes is (ayn2.7s (0)15.75 (ars (0)5.25 Answer (0) Solution . wroduction in three shift employing in three ship Panel OMS in tonnes. Panel OMS in tonnes = 284°. = 15.75 tonnes Statement for linked Answer Questions 50 and 51: Airat a density of 1.2 kg/m? flows ina straight duct such that the velocity atthe centre is 12.5 m/s, The method factor for the velocity profile is known to be 0.80. @.50 The velocity pressure value in the duct in Pa is war (a7 (60 (0)83 ‘Answer (C) Solution. Velocity pressure) =2x v8 xp = 1x (12s xos0tx12 Note ~ method factors the ratio of true velocity to the observed velocity 264 = 60Pa ast Theatr low encounters asymmetric expason such thatthe cross-sectional area of the duct becomes double, The static pressure value atthe inlet and outlet ofthe expansion are 60 Pa and 90 Pa respectively. Neglecting fiction, the shock pressure loss on account of expansion inPals ws (6)22 (38 (0) 46 Answer (A) Solution. Now, Total inlet pressure = 60 + 60 = 120 Pa ‘otal outlet pressure = 15.+90=105 Pa Therefore, ‘Shock pressure loss = 120 ~105 = 15 Pa 52 Which of the following options isthe closest in meaning tothe word below Exhert (urge 265 ass ass (@)condemn (C)restrain {0) scold Answer (a) The questions below consists ofa pat of related words followed by four pars of words, Select the pair that best expresses the relation inthe original pai. Preamble : constitution (A) amendment staw {8) prologue : play (Q)episode : serial {0) plot: story ‘Answer (8) Choose the most appropriate word from the options given below to complete the following — report, extolling only the strength ofthe proposal, (A) reasonable (0) supportive (©) biased (0) fragmented Answer (C) Choose the most appropriate word from the options given below to complete the following sentence : If the country has to achieve real prosperity that the fruits of the Progress reach al, and in equal measure. (A) inevitable (8) contingent 266 (Q)oblivious {0) imperative Answer (D) Atthe end of two 0.56 person invests Rs. 1000 at 10% annual compound interest for 2 year invested at an annual simple Interest of 12% for S years. The total years the whole amount i value of the investment finaly is: (a) 176 (@) 1760 {93920 (0) 1936, answer (0) Solution We have, Compound interest = P (1 +55] 00 [+ = Rs.1210 And simple interest = "52 = RS.726 + Total investment = 1210+ 726 = 1936 rupees 57 The ban on smoking in designated public places can save a large number of people from the well know effects of environmental tobacco smoke. Passive smoking seriously impairs ospiratory health. The ban rightly seeks to protect non- smokers from its all effects Which of the following statements best sums up the meaning of the above passage. (a) Effects of environmentat tobacco are well known, 267 {8) The ban on smoking in public places protects the non smokers (€ Passive smoking is bad for health (0) The ban on smoking in public places excludes passive smoking Answer (8) 58 Given the sequence A,8,8,6,6,6,0,0,0,0,. vetc, that is one A, two Bs, three Cs, four Ds, five sand so on, the 240" letter inthe sequence will be : wy (yu (or (ow ‘Answer (A) Solution We know, sum of n natural numbers is (nea) 2 n(n) 2240 Or n? +n —480=0 anew Therefore, 22° letter willbe V. 59 Consider the set of integers (1,23, . 5000}. The number of integers that i divisible by neither 3 nor4 is: (a) 1668 (8) 2084 ()2500 (0/2016 Answer (C) Solution. When, 5000s divided by 4 then quotient = 1250 {5000 is divided by 3 then quotient = 1666.67 5000 is divided by (4%3) =12 then quotient 16.67 268 Hence, [Number of integers that is divisible by neither 3 nor 4 = 5000 ~ (1250 + 1666.67 ~ 416.67) =2500 269 Gate Solution -2008 Q1_HAisanorthogonal matrix, then (WaTeat (ATA o newer (A) solution In case of the orthogonal matrix AAT = ATA=1 Oe a! Ina norinal (Gaussian) distribution curve, the area between one standard deviation from az mean on either side in percent is wso (es. (oss (095 Answer (8) 03. Ameasure of dispersion of a sample data set is. (A)mean—(B)mmedian (CJ mode_—_(O) standard deviation Answer (0) aa. thew ttntin, (2) wo = at (0) Non-existent 270 as. a6. qv. as. 1.) and R represent the unit vectors in the positive x, y and 2 directions ofa Cartesian coordinate system, Using the right-hand rule, x j represents ies ot wove Answer (C) ‘The rock mass classification system that considers “active stress" factor is {A)Qsystem (8) RMR torap (o)02=01 ()01 ><; 9% Oer=a2>0, (Woy = 0; > 05 Answer (8) Ina longwall mining subsidence phenomenon, the "angle of break" is the angle between (A) the vertical tine at the panel edge and line connecting the panel edge and zero subsidence. ‘on the surface (8) the vertical ine atthe panel edge and line connecting the panel edge and point of critical deformation on the surface (€) the vertical tine at the panel edge and line connecting the panel edge and the point of the ‘maximum tensile strain on the surface (0) the horizontal ine and the line connecting the panel edge and zero subsidence on the surface Answer (C). an as. a1. an. Pocket and wing technique of pillar extraction is related to {8) bord and pillar method (8) room and pillar (C) Wongawilli method (0) shortwall method Answer (A) Anon-elect Jetonating relay DOES NOT contain, {A) delay element (6) fuse head {c)metal sleeve (0) neoprene connecting tube ‘Answer (C) [Anicon ore deposit has a mean grade of 63% Fe. During the course of mining, 30% fines by ‘weight are generated at a grade of 72% Fe which are rejected. The effective mean grade of the deposit in FE percentage is waysoa (o)s3.4 (504 (yaa Answer (A) Solution au effective mean grade (Fe %) ‘Assume W fs instu tonnage of Fe deposit~ 20.720) 100 oes x 100 =59.1% Koepe system of winding does NOT include (A) tapper guide (0) limit switches {€) safety hook (0Ybrake Answer (C) am Q13, —Agasmask does NOTinclude (A) check valve (8) warning device () face piece assembly (0) coolant canister Answer (0) 14, Resuing stoping method is adopted when ore body is {8) very steep and thick (a) fat and thick (0) vory steep and thin {(G)flatand thin answer (A) ‘Moody diagram represents resistance coefficient in terms of ——L—— {c)srface tension and viscosity (0) Reynolds number and surface tenslon Answer (A) 16. Anares of 100 m? is measured ona plan having RF of 1/800. If the RF were tobe 1/2000, ‘Te area in m? would be ays (yao (1250 (0) 625 Answer (0) soution. _ Drawing demension of object ‘actual dimension of object 00m? VI00= 10m 4. _ brewing tense or width of objet Given, actual area of object Actual length or width of object 0125 m Drawing length or width of objec an au. aus. aus, Now, Hoos ~ aceaulfongh o width of obec “actual length or width of object = 25m + actual area of object = 625 m? ‘As per the DGMS norms, the severity index isa measure of (a) fatality rate (8) serious injury rate (C) numberof reportable injuries (0) accident proneness of mine Answer (0) ‘A balanced transportation problem is characterized by (A) total supply exceeds total demand {8) total demand exceeds total supply (total demand is equal to total supaly (0) total supply ic either equal te or more than total demand” Answer (¢) {nthe context of project management techniques, the TRUE statement is (A) CPM is stochastic and PERT is deterministic (8) CPM is deterministic and PERT is stochastic (C) Both CPM and PERT are deterministic (0) Both CPM and PERT are stochastic Answer (8) For mining property appraisals, typical reports prepare are Bankable Feasibility report (BFR), Conceptual Plan Report (CPR), Feasibility Report (FR) and Detailed Project Report (DPR). The shronological order for the preparation ofthese reports is (A) CPR FR BFR-+ DPR (8) BFR CPR >DPR-> FR (c) FR BFR -+ cPR-+ DPR (0)cPR- BFR + DPR FR Answer (C) 274 21, The mean of the cubes ofthe frst n natural numbers is (yearn? gymernern Ig oO Answer (A) 22, Thesumotthesgenraiesotthematix|! 2s {a)-3 {8)-1 (qa (03 Anower(@ sovaion. ‘The sum ofthe gen vale ofa mais the sum fhe clement of th pric dagonat 1021 23. Thevalue of V.Fof a vector F= 4x72 + 3xy?j + xy2%K at the point (1, 2,2) is wea (026 (930 (oa Answer) Sotation axttt Say? + yeh wae =(1 2412 42). (Outt ey? +9278) V.F = (Bx + 6xy + 3xyz?) pein (1.1.2) WP = (OKI HORI XT EIT KI) wr =26 a2. ‘The function f(x) = x3 (1 ~ x) is integrated between 0 and 1 (both inclusive) using closed form method and also by Simpson's 1/3 rule. the difference in the values obtained from these methods is tayo (8)3/a80 (c)3/120 (0) 1/20 Answer (C) Solution . [ireve= [Lea-ou A feoax Lied: = 5 Simpson's 2/3 rule [roe ean SpG = 2dr = ‘Therefore, Difference in values = + Q25. Water starts to flow into 2 sump initially containing 250 kt of water, The inflow rate of water fs 4t L/min where trefers to time elapsed in min. Ifthe pumping rate of water out of ‘the sump is 250 L/min, the total volume of water in the sump after 3 hours in KLis (Ay2505 (6) 255.6 (c)269.8 (0)2809 Answer (C) Solution. 276 ———aen 0m 1n3 hours total water that has discharged from sump ~ py = BARI = a5 KL in 3hours total water inlet into the sump. fa dem 2 2718°° 1, = 648 KL + Total water in sump after 3 hours - W, = 750.464.8485 Wy, = 269.8 KL 26. There are $0 lemon trees in a reclalmed mine area. Each tree produces 800 lemons per year. For each additional tree planted in this area, considering all trees, the output number of fruits per tree drops by 10 lemons ina year. The number of trees that to be added to the existing ‘reclaimed area in order to maximize the total number of lemons in the yea (ayo (8)35 (a6 (0/26 Answer (8) Solution 10800240000 Before planted additional tree , total number of lemon productior Let x number of trees added to maximize number of lemons, ‘Then, total number of tree= (50%x) a7 But output number of lemons drops by 10 per tree ina year. So, total number of lemons per tree in a year=(800—10%) Therefore total production (= (504) (800209) Now, = (60-+)(~10) + (800 ~ 10%) x1 formas, = 0 500 ~ 10x +800 ~ 10x ~20x x=as ‘of a dry coarse grained sandstone rock sample are 3.0 027. Thegrain density and bulk der ‘gm/cc and 2.7 gm/ce respectively. The void ratio of the sample in percentage is (a4 (8100 (ua (0/305 ‘Answer (C) Solution. We know, volume of sample~volume of grain volume of void volume of grain Void ratio volume of grain volume of sample _ Void ratio = _ volume f grain mass of sempu aensty of grain maser grain Void aio = tebe ems aensty oF praia density of grain ens of sample Void ratio = 278 sae 113% 128, The ratio of uniaxial compressive strength to uniaxla tensile strength ofa sandstone specimen is 8:1.The theoratical value of angle of internal friction of the specimen in degree is (ays. (@)an (32 7 Answer (A) Solution We have, a _Atsing o% 1-cosB 1_1tsing o=si? 229. Acircular tunnel is made underground where far field vertical and horizontal stresses are Po ‘and KPo respectively. The tangential stress (ap ) at the boundary ofthe tunnel for 0= 45° from the horizontal plane is 3Po, The value of Kis (oa (2 (3 ‘Answer (C) Solution Tangential stress (agg) = Pe[(1 +k) ~ 2 (1.~ k) cos 28} Given that, vertical stress (Pz) = Py And tangential stress (9p) = 3Py, @= 45° 3 P= Pal(-+ H)— 2(1 —K) cost x 4599] 279 ‘The bending moment diagram for the shaft shown below resembles which one of the 30, Following graphs? p04) sat Length) w © cy} see enh Answer (0) Solution 280 me Re 1. ZMp RyX4-5%2=0 Ry =25KN And also, 2V Rat Re 25 KN ‘Now, bending moment calculation 31. Amining equipment has a life of S years with no salvage value. Assuming that the depreciation of the equipment is calculated by the straight Iine method, the average annual value of equipment in percentage ofits original value is (ay20 (8) 40 (50 {) 600 Answer (A) 281 Solution. We have, {first cost~solwage value ‘Annual depreciation cost = : fein years Given that, life in yea Solvage valu ‘The average annual value (D) of equipment in percentage = LSE est (Or D=20% of frst cost (original cast) 032, Air flows at 2 m/s through a forcing fan duct of 0.3 m? having uniform cross-section, The Duct resistance is 40 Ns? mand air density Is 1.2 ka/m:. The total pressure generated by the fan is Pais (A) 186.7. (8) 160.0 (o1a33 (0)26.7 Answer (A) Solution. ‘Total pressure(P) generated by the fan = velocity pressure «static pressure Ppa xvi xp tex? Ppa txpx(2) + axa? y+ 40x 22 betax 186.7 Pa 33, Match the folowing in the context of indian mining practice: Equipment Power source P. Rocker shovel 1. Battery Locomotive 2. Compressed air Shearer 3. Electricity (maximum voltage 6.6 kV AC) 22 ‘5. Dragline (24 m? bucket capacity) 4. Eletrcity (maximum voltage 1.1 KV AC) (AP -1,0-2,8-3,5-4 (8)P-2,0-1,R-4,5-3 {C)P-2,Q-1,R-3,5-4 (0)P-1,Q-3,8-2,5-4 Answer (8) 35, The planes H and V represent the horizontal and vertical planes respectively as shown in the figure, Which one of the following Mohr circles represents the stress conditions applied in planes H and V? All stresses in MPa ‘rand oy refer shear stress and normal stresses respectively Note: shear stress is postive it tries to rotate the element in clock wise direction o ® " © g © d Answer) Solution. 283 3s. Note ‘Take point © and draw a horizontal line OX, cut off Ol and OK equal to (~ 4 MPa) and (-8 MPa) respectively. Now erect a perpendicular atJ below ine OX and cutoff J equal to negative shear stress (-2MPa), Similarly erect a perpendicular at K above OX line and cut of KE equal to positive shear stress (2mPa). join DE and bisect it at. Now with Cas centre and radius equal to CD or CE, draw Mohr circle of stresses. ‘Two splits A and B are ventilated from an intake airway. Resistances of the splits are 0.5 ‘s?m-® and 0.8 Ns*m”® respectively. A regulator Is placed in split 8 to maintain a flow of 15 ‘m3 /s and 10 m/s n splits A and B respectively, as shown inthe figure. The size of the regulator In ma? is (230° (8) 1.30 (c) 120 (oy1a3 Answer (A). 284 s. oho Solution. For split A P= RAXQy P, = 05 x 15? = 112.5 Pa For split 8 Po Po Where Reresistance of regulator Here, Py = Pa And we know, size of regulator) = 32 10m? 285 36. The concentration of OH” ion in a mine water sample is 10-1 mol/L. the pH of sample is (HO oa (oa Answer (8). Sol We have, P(OH™) = ~loglow~] (OH) = ~log(10-*] s pOHM) = 11 Since p(H*) + pOH™) = 14 att) + 1114 =p") =3 Q37. Amine having a reserve of 320 Mt produces 4 Mt of ore at the end of 1st year. Ifthe mine Increase production by 10% every year, the percentage of the reserve that still remains at the lend of 21st years (so (a) as (25 (0)20 Answer (0) Solution. Percentage of reserve (R.) that stil remains at the end of 21° year: 320-414 (1404) +401 + +O+0a7 ~ 2 z 100 f= MALL 2D" oer) 499 286 038, Match the following: ore, rock strength Mining method a, Sublevel stoping room and pillar «Block caving (0)P-1-b,Q-2-6R-4-a (0)P-1-¢,Q-1-b,R-2-a ‘Type of ore deposit P.flat, thin 2, Strong, strong, Q.Massive 2. Weak, weak R. Steep, thick (a)P-1-6,Q-1-3,R-2-b (OP-2-b,Q-1-a,R-1-¢ Answer (8) 39, Match the following: ‘Stoping method Advance of stoping face 1. Sideward vertical slices P. Shrinkage stoping 2.Upward horizontal tice Rill stoping R.Blasthole stoping 3.Downward horizontal slice S. top slicing 4, Sideward inclined slices (A)P3,04,R4,52 (8)P2, 03, RASH (C)P-2, 0-4, Ra, $3 (0)P4,03,R2,54 Answer (C) 40, Which one ofthe following eraphs typically represents the standard strain-time creep bohavior of an isotropic rack material under constant temperature? P, Sand T in the figure refer to primary creep, secondary creep and tertiary creep respectively {Capt 287 Tee (Groh 1) cGnph tv) (A)Grapht (8) Graph {Graph (0) Graph Answer (A) 41, The following data represent the number of workers suffering from pneumokoniosis in 10 coal mines. Mine [1 [a [um Tw Tv Tw Tw Tv [x [x Number] 10 [16 [ia [as [a4 [2 [ar [a3 fas [ae ‘The number of mines falling above the 50th percentile in terms of the number of workers Suffering from pneumokoniosis is. (2 () 3 os Or ‘Answer (D) Mine (Mp Number(N) Cumulative Number (CN) 1 10 10 " 16 6 ” 4 40 wv 15 55 v 4 eo w 2 a1 vu v 98 288 vu 2 am a5 126 x 2 8 ' Now, or 50! pecentie= 22 «69 So, number of mines falling above the 50" percentil 42. Cause -wise data for injures in an underground coal mine fora five year period is given below: ‘Cause of inpry ‘Number of injuries Fall ofroot 27 Fall of person. 22 Rope haulage 7 Explosive 3 } ‘Other causes a The cumulative probability of injury due to fall of roof and fall of person is (ajoss (80.50 (9036 (o)0.29 ‘Answer (A) Solution Cumulative probability of injury rate due to root fall and fall of person is— area, “DRT ESSE = 065 Q43. Consider the following linear programming problem: Maximize r= 3x42y Subjectto — 3x+2y 215, 2x4 3yS6 x20,y20 The above linear programming problem has {A) unique optimal solution (8) multiple optimal solutions. (C) unbounded solution (0) infeasible solut 289 Answer (0) Solution. ven linear programming has infeasible solution Infeasible solution n linear programming, there is no common shaded area 44, Amine workshop has 4 lathe machines and 4 tasks for competion, Each ofthe machines can be perform each of the tats Each task can be assigned to one and only one machine, Estimate cost in rupees to complete each task i given inthe matrix below. Machine he Th Hz ([® Tk Te @ [a a fe maps |e [es jo] |r ‘The total optimum cost in Rupees for assigning the tasks to the machines is. (a) 210 (8) 215 {c) 220 (0) 296 Answer (C) Solution 290 0 7 7 48 To 12 33 24 Task Machine Tosk 291 Machine My aM, My, hoa as 0 43 oe o] a5 4 xf] > 2 as m7 ° a ° Hence, Total optimum cost is =$2-+49+47 +72 220 rupees Q45. 1100, 3 © power supply system of a mine draws a load of 185 kW. The ammeter reading shows 115 a. The power factor of the system is (joss (@)0.73 (qo6a (0) 0.48 Answer (A) Solution. We have, Power= V3xV; x I, x cos 6 185 x 10° = V3x 1100 x 115 x cos #080 = Fy rons 2» cosd 184 46, Two belt conveyors load a ground bunker, each ata rate of 400 tph, which is intially filled with 10000 t of coal. Coa is discharged from the bottom of the ground bunker onto belt conveyor ata rate of 1200 tph. The time elapsed in hours before the bottom conveyor starts to operate below its rated capacity is 292 (es (ayes tors (0)25.0 ‘Answer (0) Solution. Here , 400 tonne of coal is produced from inital filled coal in the bunker to maintain Aischarge rate 1200 tph on bottom belt conveyor. So, the time(T til belt conveyor works ats rated capacity: T 25 hours 47, The case flow table of manganese mine for a particular year Is shown be Tem “Amount (Rs. in las) Revenue, 900 ‘Cost (other then depreciation) 300 Depreciation 100 Profitbefore tax 500, {the corporate taxi 50% of the Profit before tax, the operating cash inflow in lakhs of Rupees is (a) 400 (®)350 © (@)250 (0) 200 ‘Answer (C) Solution 293 as, 4s, Note - corporate taxis imposed on net profit. trem amount (Rs. n ks) Revenue 900 Cost (other than depreciation} 300 Depreciation 100 Profit before tax 00 + corporate tax = 2% 500 = 250 aks 00 — 250 operating cash flow 50 lakhs Ian area within a surface mine, under static condition the following gases are found: N02, Cz, 03 and 502. Assuming no diffusion, reaction and bonding ofthe gases, the ‘concentration of the gases from bottom upwards will be in the order of (8) N02 C02, 03 and Ox {8)502,N0,,COzand 0 {C) $02, 04,NO2 and CO, (0) NO2,COz, $03 and O5 Answer (C) Ina mine site, the cost of shaft sinking in lakhs of Rupees is given as 2.640 + 34.8, where D Is the shaft depth in m. In the same. 2, the corresponding cost of driving an incline is ‘0.961, where Lis the length ofthe incline in m. Assuming by D rat {5 3.0, the depthinm beyond which the shaft, sinking becomes more economical is fayas (a) a8 (1a, (0) 155 Answer (C) Solution. 294 os Zz Atbreakeven point, G=G 2.640 + 34.8 = 0.961. D D=145m Q50. Match the following: Seam characteristics Coal mining method 1, Mechanized fongwall . 12 m thick flat seam Q.7 m thick seam at 65° inclination 2 Descending shield 3. Mechanized integral caving R. 3 m thick flat seam S.7 m thick seam at 25 inclination _ 4, Jankowice (A) P-4,Q-3,R-2,5-1 ()P-3,Q-4,R-1,S-2 (0)P-3,Q-2,R-1,S-4 (©)P-2,.Q-3,R-4,5-1 Common data for questions 51 and 52: ‘Workimen arrive at a mine workshop to receive tools for maintenance . The Inter-arrival time of workmen at service counter is exponentially distributed with an average time of, 10 min, the service time at the counter is also distributed exponentially with a mean time 295 16min. 51. Probably tht tere isa queue (more than one workman) a the service counteris woz (e038 (cjoaa(o)as0 Answer (D) Soliton. ara te 6 Anvatete And service time : 6 min, Service rte = j= 2 per min. Thereore, Probably that heres a queue (more than one workmen) a he service counter = trafic ltensty or uation aif) 52, _ Average time spent bya workman wating for hs turn tbe served in mini Wo (cas (ie ‘Answer (C) Solution ‘Average time spent (T) by a workmen waiting for his turn to be served = ‘Common Dat stions 53 a Atacheometeris set up ata station 8. The RL of the station B is 150 m and above the MSL. by holding staff vertically ata station ‘A’, the following readings are taken, 296 {eerticalangle | _staffreadings(m) upper 536 toe | mide | ‘The multiplying factor and adltive constant of the instrument are 100 and 1.9 m respectively 1053, The horizontal distance between the stations A and B in mis (ajasas§—(8)366.3 (c)409.4 (0/4575 answer ( Solution. ae aK TEE Lis <———_—— >» ———— Multiplying factor= 100, and $=5.36 ~0.80= 4.56 m Additive constant = 1.9m we have, D = KS (cos)? + (f + d)00s8 D = 100 x 4.56 x (cos26°36)* + 1.9 x (cos26°36) 166.298 m 297 Q51. Ifthe height ofthe instrument is 1.2 m, the RL of the station ‘A’ above the MSL is m is (4)337.6 (a)3345 (3315 (03303 Answer (¢) Solution . We have, = KS sind cos + (F +d) sing V = 100 x 4.56 x (sin 26°36) x (cos 26°36) + 1.9 x (sin 26936) ov 183.45 m = R.Lofstation A above MS = 150412 + 18345 — 3.08 Common Data for Questions 55 and 56: A turbine pump of efficiency 70% discharges water atthe rate of 2100 L/min ata total head of 100m. 55, the pump is un by 3 motor of efceney 9%, the input power required forthe motor ia wis (a)z249 (9031 (gana —_(o)s450 Answer (0) solution. Pogo Input power Pro metor =P 5 10° P= p.g.h = 1000% 100 x981 = 961000 Pa @ = 2100-4, = 0.035 m/sec eer ean re fd "inp power (P) 10 moto 0.90 545 kW 298 056, Ifthe velocity of water in sucti ‘and dolivery pipes ofthe pump are 1.8 m/s and 2.5 m/s respectively, the diameter of suction and delivery pipes in cm are (4) 15.73 and 13.35 (0) 7.86 and 6.67 (6) 5.78 and 6.02 (0) 4.97, 4.22 Answer (A) Solution. Weave, Q= AM = Any L Q= 2100 = 0.035 m*/sec ¥, = 1.8m/s0e, ¥, = 2.5 m/sec Now, Q = Ai¥s_ [inthe suction side} 0.035 = 1.8 x A or 4, = 288 Or Ex a? = dy = 0.1873 m = 15.73 em And in the delivery side- @ 0.035 a 1335 m = 1335.em Statement for Linked Answer Questions 57 and $8: A fan running ata speed of 280 rpm circulates 305 m/s of airin a mine, 57. Ifthe power input to the motor for driving the fan Is recorded to be 75 kW, with the combined efficiency of fan and motor at 70%, the fan pressure in Pais (a) 50 (a)350—(c) S00, (0}650 299 ‘Answer (€) Solution. Weave, Input power (0 motor= => P2— x 10°? ki 75 = P28 10 500 Pa 58, Ifthe fan pressure sto be increased by 200 Pa by changing the fan speed, the fan speed in +m wil become (ay768 (8) 549 (c)392 {0)332 Answer (D) Solution. Weave, pressure « (speed)? Gy P, = 500pa 500 + 200 = 700 pa 32 rpm ‘Statement for Linked Answer Questions 59 and 60: ‘surface mine blast design has 9 holes ina row, each of 8 m length and 200 mm diameter. the spacing and burden are 6 and 5 m respectively. The length of subgrade drilling is 1 m and 300 the density of in-situ rock is 2.43 t/m?, 59, Assuming no back break, the output per blast ints (a) 4593 (8)5905 (cy 6124 (0) 6299 Answer (A) Solution. We have, ‘output per blast = No.of holes x spacing x burden x density x (length of holes ~ subgrade length) = 9x 6x5 x(B~1) x243 = 4593 tonnes of 0.9 t/m? and stemming length of 2m, the powder factor (G60, Considering an explosive de from the bast in t/kg (4) 400 (aaz (aor, (386, Answer (0) Solution. Explosive required per round of blasting = cross sectional area of hole x No.of hole x (length of hole ~ stemming height) x explosive density = 4x 028 «9x (@—2) x 09x 1000 = 1526.04 kg ‘esive required (9) + Powder Factor = 3.01 t/kg 301 Gate Solution-2008 1. Thetrace of the matrixis 223 fa 3] 4 1 2 we (0)7 (a8 (0/9 Answer (A) Solution Note: Trace of the matrix - sum of diagonal elements ofa square matrix is called trace of 1 Trace of the mate =2+242= Q2. _IfXisa continuous random variable and f(x) defines its probability density function , then the expected value of Kis WEZEfade (8) SY ()E2axef) 0) F2ay@dax Answer (D) 3. Thetoolused to correct borehole deviation is (A)stringshot (8) Kelly (C) Whipstock (0) Rachet Answer (C) 4, Aphreatic surface experiences a pressure (A) tess than atmospheric pressure (8) equal to atmospheric pressure (C) more than barometric pressure (0) ess than barometric pressure 302 ‘Answer (8) Q5. The oad yield characteristics of a hydraulic prop is represented by the curve / /n — (a) (8) _ o Oy Answer (A) 6, _Inlongwall caving, the thickness of immediate roof s calculated from (A)Bulking factor and width of the longwall face (8) Sear thickness and width of the longwall face {(C) Seam thickness and bulking factor {0) Seam thickness and length ofthe panel ‘Answer (C) 303 7. During over-winding, a cage is safely suspended in the headgrear due to (4) Bulle chain (8) Rope capel (€)Dstink (0) detaching hook Answer (0) 8. Depending upon the decreasing ability of surrounding rock to store strain energy, the underground metal mining method ean be ordered as (a) cut and fil stoping, sublevet caving, sublevel open stoping, block caving (6) Sublevel open stoping, cut and fil stoping, sublevel caving, block caving (€) Subteve caving, sublevel open stoping, cut and fillstoping, block caving (0) Block caving, sublevel caving, sublevel open stoping, cut and fl stoping Answer (8) 89, Ifthe swell factor of ore in shrinkage stoping is 4.4. the output from the stope in percentage of broken ore is wo (8)29 (40 {0) 100 Answer (8) Solution extra volume Broken ore volume x 100 %of broken ore Q10. The velocity of the wave type that determines the ‘rippabilty’ of rackmass is (Appavave —(B)S-wave(C)Raleighwave —_(D) Love wave 308 Answer (A) 11, Inthe order ofthe chronological development, the longwall support system are arranged as P. Powered support Q.tinkbar and friction support A. Frame support 5. Hydraulic support () P>Q>R>s (B)R>S>Q>P (QS>R>P>Q_—(DJQ>S>R>P ‘Answer (8) 12, Effective temperature is estimated from (8) Wet bulb temperature ,relative humidity and aie velocity (8) Dry bull temperature, ralative humidity and aie velocity (C) bry bulb temperature, wet bulb temperature and ai velocity | (0) Dry bulb temperature, wet bulb temperature and relative humidity Answer (C) 13. Pressure -quantity characteristics ofa mine fan is given below: f| LoL ‘The combined characteristics of two such identical fan installed in parallel is 305 «@ } LE | omy uray : ) (0) ‘Answer (A) 14 —_Underidentical water head and roadways candition for water dam construction, if P,Q.and R represent the thickness of flat dem cylindrical dam and spherical dam respectively. The thickness would follow the order (AyR>P>Q_— (8) P>R>Q = (C)P>ODR (0) Q>P>R Answer (C) 125. The grain size distelbution of sol is known as (A)Peemeabitity (B)Structure _(C) Porosity _—{(D) Texture Solution (8) 306 16. Electrostatic precipitator works on the principle of (Alcapacitance change (8) Ionization of particles (€) Electro heating of gases (0) Centrifuging the gaseous molecules Answer (8) 17. Inthe figure shown below , the distance RP and PS are measured to be 80 m and 72m respectively. The distance PQ in m (a)eo.a (8) 65.4 (608 (0) 68.4 Answer (C) Soluti 307 tan 2PRS = n 30 2 2PRS = 41.99" ZPQR = 180° ~ 90° — 41.98" 2PQR = 48.02" Now, In APSQ tan 49.02? = 2 “7g PQ = 648m 18, In PERT network, the activity duration is assumed to follow (a)Beta distribution (8) Binomial distribution (C)Normal distribution (0) Weilbul distribution Answer (A) 19, Foran? problem, identify the INCORRECT statement {A) Optimal points ies in one ofthe corner points (0) Objective function is near 308 {CVA the constraints are linear (0) optimal point ies in any ofthe interior points ofthe feasible region ‘Answer (0) 20. Inn bi-axial stress field the vertical stress is 10 MPa and the poisson ratio forthe rock mass 150.2. The horizontal stress in MPa is, was (25 (920 (05.0 Answer (8) Weave, horizontal stress _ poisson ratio vertical stress 1=poisson ratio horizontal stress 02 70 = Toa “horizontal stress = 2.5 MPa a2, Gens Bench eight -12m bucdon Ar Spacng-Sm Sub- rade lig-2 fsooseper hole 12045 | Density of rack - 2600 kg/m’ | The ponder factor intone I w20 was (asa 7s Answer (C) 309 We hay blasted material (tonne) Powder Factor “explosive required (kg) hole length xspacindxburdenxrock density (tonne) ‘explosive required (kg) 1245 x 2600/1000 52 120 222, Match the Equipment slice thickness (rangeinm) Action P. Dragline 2.6.12 a. crowding @. shovel 2.30-40 bs. Hotsting R.Surface miner 3.02-0.4 «Cutting (4) Pa; a2aRse (P24; A1a;RI< (0) P29 arbnse (0)) Pah; R103 Answer (8) 23. Giver Value of ore s.600 per tonne Production cost Rs.600 per tonne Cost of overburden removal : Rs. 50 perm? ‘The break-even stripping ration m/tonne is aa 3a (as (0) 14 Answer (A) Solution. 310 We know, Break even stripping ratio C production cost per tonne of ore 0st of extraction per cubic meter of overburden material 24. Four mines A, 8, Cand D are located along a road as shown with produetion in Mt per year 4, 2,1.and 3 respectively. n order to handle total coal produced, the distance of the coal ‘washery along the road from the mine A in km is a 8 c cy ° tem 2km km (a) 401 (e)3.91 (3.81 (03.72 ‘Answer (0) Solution Diam . Where, x- ideal distance from coal washery from mine A # Xx1#2x (=I) 1x (e-3)=3x (7-H) RAD 24x-3=21—¥ ex =37Lkm sit 25, A shaft insets as shown below. To transport a 15 m long object, the height inmshould be o-7m 4 jos (8)7.0 16s (0/59 Answer (8) Solution. 312 in aaBe And BC = JCS = 0.608} = 10.606 m In AABC & ACDE 026. Match the following Blast problem cause P. Misfire 41. Poor stemming vibration 2.Low current R, Blown -out-shot 3. Excess charge 5. Cutoff shot 4. Improper delays (A) P23, 02, Re, St (8)P-4,04,R2,53 (0) P2,03,R4,54 {0)P-1, 02, R453 ‘Answer (C) 227, From the stress-strain diagram shown below, the tangent and che secant moduli of elasticity in GPa are 313 2 7 = % 2 k 30 2030 33 strain (eer. units) (a) 4022 (3323 (93315 (oaoas Answer (A) n 0 z 4 20 a 23033 strain (micro units) ‘Tangent - tangent to @ plane curve ata given points the straight line that touches the curve 34 a2. Solution. at that point. Secant- secant toa curve a line that intersect two point on the curve, In above figure CD is tangent and ABis secant Hence, ‘A bord and pillar operatios ia planned at a depth of 300 m in astrata of average unit, ‘weight 28.5 kN/m? and compressive strength 15.50 MPa, Ifthe width of opening is 6 m considering a factor of safety of 1, the maximum possible extraction ratio in percentage is (a)2a (634 (as (0)53 Answer (0) strength of pla = 15.5 MPa = 15.5% 108 N/m? Depth = 200m Unit weight (w) = 24.5 kN/m! = 28.5 108 N/m? Wicth of gallery or opening (b) = 6 m strength of pillar Factor of safety = rating on pillar Andi cas of square plo oat acting on ptr = w. 0 SEP 155 x 108 oP 185 x 106 245 x 109 x 20083 (On solving, we found that a2. 930, 13.268 m Now, Extraction ratio (96) = 1— GE x 100 132608 - ix Gazcneae* 100 = 53% Match the following Stoping method Ore handling system Support system P. Breast stoping to 2.Insitupitar . cutand fi 2 Scraper ». Unsupported A. Sublevel stoping 3. Gravity flow Mil talings (4) P29; ae (P49; a3¢R2 (0) 2m; arena (0)P-14; O33; R26 Answer (A) Match the following Access Moulage Mineralisation location P. Shatt 1. Track 2 moderate depth @. decine 2-Trackess Deep seated R. Adit 3H .titock (A) Pata; a3.b; RI (0) Pa; a2a;Re (0) Pra; Qa-b: RI Answer (8) (0) P.2-; Q3-¢; Ra 316 Q31. Match the following ‘Mining method Operation P. Bord and pillar 4 Longhole radial diting 1 Sublevel caving 2. Splitting and slicing R. Longwall retreating 3. Loosening under strata pressure S. Integrated Caving 4, Mechanical cutting (a)P-a, a4, R3, 5-2 (8)P-2,03,R-4,5-4 (0) 94,02, R3, 54 (0)P2, 03,4, 53 Answer (0) 32, A30mtape has an error of 0.005 m. Ifa length of 1500 mis measured with this tape, the ‘expected total error made in the measurement in mis (a) 40.025 (8) +0.030 (0) + 0.035 (0) £0.04 Answer (C) Solution, The expected total error made inthe measurements = + 0,005 x | = £0035 033, Match the following Instrument Principal features Application Ting level 4. Micrometer a. Levelling ©. Microoptictheodolite 2. Magnetic needle b. Traversing R.Telescopicalidade 3, U-tube Aalmuth 5. Compass 4 Plane table surveying 4. Contouring 37 (A) Pb; O26) R4asad (8) Pah; 3a; R4-cS2d (c) P2< 08 Roasted (oP. 9) QA-b; RAGS ZC Answer (D) 34, confined aquifer of 75 m thickness has 2 monitoring wells spaced 2500 m apart along the direction of water flow. The hydraulic eonductivity of the aquifer is 40 m per day. The water head difference between the wells s 1.5 m. applying the Darcy law's, the rate of flow per minute of distance perpendicular to the direction of flow in m*/day is (2a was (14s (on2 Answer (8) Solution, We have from Darcy law, Qe kia where, hydraulic conductivity = 40 m/day AA- Cross sectional area Ihde gaiens = 2 Ih difference of hydraulic head over distance L] 318 Hence, a= 40x 2x75 Bm? /day 035. Precipitation of the metalic ion in mine water drainage Is carried out by (a) Cas0,and mgso, (8) Cacoyand mgCOs (€) Ca(OH) and NaoH (.Caco;and mgs0, Answer (0) 36. Adrum winder of radius 2.5 m draws. power of 308 KW when the maximum roe speed is 7 m/s. The RMS torque in Nm is ss (075 (© 10 (0) 104 Answer (C) Solution. Powers force x velocity 308 x 1000 fort «The AMS torque in kN = force x radius = 44000 « 2.5 = 110 kN, 37. Abelt conveyor conveys material of average cross-sectional area of 0.09 mn? Of bulk density 15 toone/m® ,at aspeed 2 m/s. The catryi of the belt in tonne/hr is (972 (0) 364 (o) 732 (0) 643 ‘Answer (A) 319 Solution Carrying capacity (1) of belt conveyor i given by Ts aby tonne/s Where, 1 =cross sectional are = bulk density =15 tonne/n® ve2 m/s T=009%15%2 1=027 tonne/s T2972 tonne/hour Q38, The wt % of solids ina sand-water stowing pipe is 60 Ifthe solids density 3000 kg/m® , the pulp density of tne slurry tn ke/mn? (ay1380 (a) 1420 fas _ ‘Answer (D) —— ‘We have, Pulp density = ae Pulp density = ass of pulp, Volume of water + Volume of sand mass of putp Or pulp density = “wasrapwarer_,mastayaame ensty of water * density of sand Mass of sand = mass of pulp x 60 = mass of pulp 06 Mass of wate mass of pulp X (100~60}% = mass of pulp x 0.4 mass of pulp «Pulp density = mass of pulp sal 320 Pulp density = 1666.7 kg/m® 39. _Amining project comprising of A,8 and C activities is scheduled for 90 days at a cost of Rs 11200 million, The manager of project decides to reduce the time for completion ofthe project 10.85 days. The decision was taken after 45 days. a 8 c a activity A 8 c Duration (days) | 49 Fo 8 ‘crashing cost [day wf a {million rupees) * ‘The minimum project cost in mili rupees after erashing by 5 days is- (a)1100— (8) 1300 (1475 (0) 1825 Answer (8) Solution Here, crashing of project is dane along C activity duration because having minimum crashing cost as comparaed to 8 activity duration minimum cost of project after $ days crashing is = 1200 + Sx 24 300 milion rupees | 040. the following information is provided for an ore deposit Number of waste blocks =10 ‘Number of ore blocks 5 ‘Volume of each waste block? = 600 ‘Total cost of waste handling perm? =. 100 32 Tonnage of each ore block Total cost of ore handling per ton Sole price of ore per ton The net cash flow of mining the deposit in lakhs of rupees, i waa (25 (9 10 (p08 Answer (€) Solution Sale price of ore = 5 x 400 x 500 = 1000000 rupees ‘otal cost of waste handling = 10 x 600 x 100 690000 rupees Total cost of ore hansling= 5 x 150 x 400 = 300000 rupees Hence, net cash flow of mining deposit = 1000000 ~ 600000 ~ 300000 = 100000 rupees lakh rupees 41, Determine the correctness or otherwise ofthe following Assertion [a] and the reason [1] ‘Assertion : While stonedust barrier may be effective against a coal dust explosion, the same isnot tue I case of firedamp explosion Reason : In general, firedamp explosion are much more powerful than coal dust explosions, (A) Both [a} and {ear false (@) Lalis true but [e Is false (©) Both [a] and [r are true and {ris the correct reason for [a], (0) Both (a] and [e] are true but [ris not the correct reason for [a] ‘Answer (8) 42 Mateh the following Component of flame safetylamp Purpose of component P. Asbestos rings 1. Dissipation of heat of flue gases, 0. Wire gauges 2, Formation of airtight joints an R. Outer glass '. Combustion chimney (a)P2,04,R3,5-4 (0) P2,0.4,R3,54 ‘Answer (A) 3. Arrest of explosion inside the lamp 4. Separation of inlet air from flue gases (8)P-4,04,R2,53 (0)P1,0.2,R4,53 43, road header district produces 20 mg/m? of airborne dust with the following size distribution Skee up to cumulative wt % am Sym 5 roy 10 en 20 so vm 50 > 50 um 100 The concentration of respirable fraction of dust In mg/m’ Is (yor (02.0 Answer (6) Solution (100 (0)10 Respirable site of dust £10 jum Hence, concentration of respirable ration dust= 20 1, gin? 323 aa, For a person working in atmosphere containing 21 % 02, the exhaled a and 16 96 0;. The respiratory quotient of breathing is (aot (8) 09 (©) 028 (0) 1a Answer (8) Solution. as. Solution 46. Solution We know that, on produced Respiratory quotient = 2 22 ‘Total number of injuries in an opencast coal mine employing 800 persons is 16 na year . AS per DGMS norms, the injury rate per 1000 persons employed Is (1a (15 (920 (0)25 ‘Answer (C) 2&x 1000 Injury rate per 1000 persons employe 20 ‘The coefficient of fiction between the tub - wheel and haulage track is 1/V3 . for the applicability of direct haulage, minimum inclination (in degrees ) of track is tayo (55 (c) 35 (0)30 Answer (0) Weave, 324

You might also like